You are on page 1of 52

` 2

Quantitative Ability & Data Interpretation


1.

Albela, Bob and Chulbul have to read a document of seventy eight pages and make a presentation next
day. They realize that the article is difficult to understand and they would require team work to finish the
assignment. Albela can read a page in 2 minutes, Bob in 3 minutes, and Chulbul in 4 minutes. If they
divide the article into 3 parts so that all three of them spend the equal amount of time on the article, the
number of pages that Bob should read is
1. 24

2.

3. 26

4. 27

5. 28

The taxis plying in Wasseypur have the following fare structure: Rs 20 for the first two kilometers, Rs 5 for
every km in excess of 2 km and up to 10 km, and Rs 8 for every km in excess of 10 km. Bullock carts on
the other hand charge Rs 2 per km. Sardar Khan takes a taxi from the Wasseypur railway station to his
home. On the way, at a distance of 14 km from the railway station, he meets Faizal Khan, and gets down
from the taxi to talk to him. Later he takes a bullock cart to reach his home. He spends a total of Rs 102 to
reach his home from the railway station. How far is his home from the railway station?
1. 17

3.

2. 25

2. 18

3. 19

4. 20

5. 21

Consider the expression


(a2 + a + 1) (b2 + b + 1) (c2 + c + 1) (d2 + d + 1) (e2 + e + 1)
abcde
where a, b, c, d and e are positive numbers. The minimum value of the expression is
1. 3

4.

2. 120 minutes

5. 243

3. 140 minutes

4. 150 minutes

5. None of the above

2. 125000
3. 250000
5. Cannot be answered from the given data

Mr. Mehra is planning for higher education expenses of his two sons aged 15 and 12. He plans to divide
Rs 15 lakhs in two equal parts and invest in two different plans such that his sons may have access to Rs
21 lakhs each when they reach the age of 21. He is looking for plans that will give him a simple interest
per annum. The rates of interest of the plans for his younger son and his elder son should be
1. 5% and 7.5% respectively
3. 10% and 15% respectively
5. 20% and 30% respectively

7.

4. 100

At the centre of a city's municipal park there is a large circular pool. A fish is released in the water at the
edge of the pool. The fish swims north for 300 feet before it hits the edge of the pool. It then turns east
and swims for 400 feet before hitting the edge again. What is the area of the pool?
1. 62500
4. 500000

6.

3. 10

Prof. Mandal walks to the market and comes back in an auto. It takes him 90 minutes to makes the round
trip. If he takes an auto both ways it takes him 30 minutes. On a Sunday, he decided to walk both ways.
How long would it take him?
1. 100 minutes

5.

2. 1

2. 8% and 12% respectively


4. 4. 15 % and 22.5% respectively

The mean of six positive integers is 15. The median is 18, and the only mode of the integers is less than
18. The maximum possible value of the largest of the six integers is

` 3
8.

1. 26
2. 28
3. 30
4. 32
5. 34
Last year Mr. Basu bought two scooters. This year he sold both of them for Rs. 30,000 each. On one, he
earned 20% profit, and on the other he made a 20% loss. What was his net profit or loss?
1. He gained less than Rs. 2000
3. He lost less than Rs. 2000

9.

2. He gained more than Rs. 2000


4. He lost more than Rs. 2000

In an examination, the average marks obtained by students who passed was x%, while the average of
those who failed was y%. The average marks of all students taking the exam was z%. Find in terms of x,
y and z, the percentage of students taking the exam who failed.
1. (z-x) 100/(y-x)
3. (y-x) 100/(z-y)

2. (x-z) 100/(y-z)
4. (y-z) 100/(x-z)

DIRECTIONS for questions 10 to 11: Answer the questions based on the data given below.
10.

Arun has to go to the country of Ten to work on a series of tasks for which he must get a permit from the
Government of Ten. Once the permit is issued, Arun can enter the country within ten days of the date of
issuance of the permit. Once Arun enters Ten, he can stay for a maximum of ten days. Each of the tasks
has a priority, and takes a certain number of days to complete. Arun cannot work on more than one task
at a time. The following table gives the details of the priority and the number of days required for each
task.
Task
T1

Priority
1

Number of Days Required


3

T2
T3

2
5

5
3

T4

T5

Arun's first priority is to complete as many tasks as possible, and then try to complete the higher priority tasks. His
last priority is to go back as soon as possible.
The tasks that Arun should try to complete are:
1. T1 and T2
3. T1, T4, and T5
11.

2. T1, T2 and T5
4. T1, T2 and T4

5. T1, T3 and T4

However, Arun's manager has told him to do some background research on the tasks before leaving for
Ten. At the same time, there is no guarantee that the Government of Ten will give the permit to Arun.
Background research involves substantial costs, and therefore Arun has decided not to start his
background research without getting the permit.
The following table gives the details of the priority, the number of days required for each task and the
number of days required for background research on each task.
Task
T1
T2
T3

Priority
1
2
5

Number of Days Required


3
5
3

No. of Days Required for Background Research


3
5
2

T4

T5

` 4
Arun's first priority is to complete as many tasks as possible, and then try to complete the higher priority
tasks. His last priority is to go back as soon as possible within ten days.
The tasks that Arun should try to complete are:
1. T1, T2, and T3
3. T1, T2 and T4
12.

1
3

2.

2. 214

1
5

3. 0.75 and <1.0

4. 1.0 and <1.25

5. 1.25

4. 12

5. 14

3. 220

4. 240

5. 248

p, q and r are three non-negative integers such that p + q + r = 10. The maximum value of pq + qr + pr+
pqr is
2. 50 and <60

3. 60 and <70

4. 70 and <80

5. 80 and <90

A number is interesting if on adding the sum of the digits of the number and the product of the digits of the
number, the result is equal to the number. What fraction of numbers between 10 and 100 (both 10 and
100 included) is interesting?
2. 0.11
5. None of the above

3. 0.16

70% of the students who joined XLRI last year play football, 75% play cricket, 80% play basketball and
85% play carrom. The minimum percentage of students who play all four games is:
1. 5%
4. 20%

19.

4.

3. 10

1. 0.1
4. 0.22
18.

3
5

How many whole numbers between 100 and 800 contain the digit 2?

1. 40 and <50
17.

2. 0.5 and <0.75

2. 8

1. 200

16.

3.

In a square PQRS, A and B are two points on PS and SR such that PA = 2AS, and RB = 2BS. If PQ = 6,
the area of the triangle ABQ is (is it a repeat)
1. 6

15.

2
5

Six playing cards are lying face down on a table, two of them are kings. Two cards are drawn at random.
Let a denote the probability that at least one of the cards drawn is a king, and b denote the probability of
not drawing a king. The ratio a/b is
1. 0.25 and <0.5

14.

5. T1, T4 and T5

Three circles A, B and C have a common center O. A is the inner circle, B middle circle and C is outer
circle. The radius of the outer circle C, OP cuts the inner circle at X and middle circle at Y such that OX =
XY = YP. The ratio of the area of the region between the inner and middle circles to the area of the region
between the middle and outer circle is:
1.

13.

2. T1, T2 and T5
4. T1, T3 and T4

2. 10%
5. None of the above

3. 15%

p and q are positive numbers such that pq = qp, and q = 9p. The value of p is
1.

2.

3.

4.

5.

` 5
20.

Ram, Shyam and Hari went out for a 100 km journey. Ram and Hari started the journey in Ram's car at
the rate of 25 kmph, while Shyam walked at 5 kmph. After sometime, Hari got off and started walking at
the rate of 5kmph and Ram went back to pick up Shyam. All three reached the destination
simultaneously. The number of hours required for the trip was:
1. 8

21.

2. 7

5. 4

2. 12.5 metres

3. 14 metres

4. 15 metres

5. 16 metres

In the country of Four, there are four cities, A, B, C and D. B is to the East of A, C is to the South of B, D
is to the West of C, and A is to the North of D. The Government of Four is planning to connect these four
cities by road such that it is possible for a person to go from a city to any of the other three cities. At the
same time, the Government wants to ensure that the total road length is minimum. The distances
between A to B, B to C, C to D and D to A are all equal to 10 km. What should be the total length of the
road?
1. 25.64 km

23.

4. 5

The central park of the city is 40 metres long and 30 metres wide. The mayor wants to construct two
roads of equal width in the park such that the roads intersect each other at right angles and the diagonals
of the park are also the diagonals of the small rectangle formed at the intersection of the two roads.
Further, the mayor wants that the area of the two roads to be equal to the remaining area of the park.
What should be the width of the roads?
1. 10 metres

22.

3. 6

2. 27.32 km

3. 28.30 km

4. 30 km

5. 36 km

Please read the following sentences carefully:


I. 103 and 7 are the only prime factors of 1000027
II. 6 6! 7 7!
III. If I travel one half of my journey at an average speed of x km/h, it will be impossible for me to attain an
average speed of 2x km/h for the entire journey.
1. All the statements are correct
2. Only Statement II is correct
3. Only Statement III is correct
4. Both statements I and II are correct
5. Both statements I and III are correct

DIRECTIONS for questions 24 to 26: Answer the questions based on the following information.
Given below are a few data points on the Indian economy from 2005 to 2010:
Indicator
GDP, current prices

Unit

2005

2006

2007

2008

2009

2010

Rs Billions 35662.2 41159.73 47675.86 54470.27 60712.76 73555.34

GDP per capita, current prices

Rs

32128.1 36553.93 41747.69 47038.23 51714.45

61784

Gross national savings Percent of GDP

32.88

34.28

36.65

32.17

35.08

32.14

Inflation, average consumer prices Index

Index

115.67

122.92

130.75

141.67

157.08

175.92

Volume of imports of goods and services % change

17.99

9.438

16.3

10.84

8.321

16.49

Volume of exports of goods and services % change

18.88

13.83

17.13

10.63

0.813

21.86

Unemployment rate

9.2

8.9

7.8

7.2

6.8

7.32

Current account balance Percent of GDP

-1.272

-1.024

-0.701

-2.475

-2.066

-3.268

* Per capita GDP is arrived by dividing GDP by population.

` 6
24.

What is the ratio of the current account balance in 2010 to the current account balance in 2005?
1. 0.35

25.

2. 4.56

3. 5.01

4. 2.57

5. 5.30

Read the statements given below:


A. Exports were more than imports in 2006
B. Imports were more than exports in 2009
C. Exports increased at faster rate than imports during the period 2005 to 2010
Which of the above statements is necessarily true?
1. A and B

26.

2. A, B and C

3. C only

4. B Only

5. A only

What was the approximate number of unemployed persons in 2006?


1. A. 100 million

2. 102 million

3. 98 million

4. 105 million

5. 104 million

DIRECTIONS for questions 27 to 30: Answer the questions based on the following information.
Data on an ongoing football league of a country is given below. 20 teams are playing in the league. The
rules of the league are as follows:
1. Each team plays all the other teams twice, once in its home ground, and once in the opponent's home
ground. These matches are known as the "Home" match and the "Away" match respectively.
2. A win results in 3 points, a draw in 1 point, and a loss in 0 point for the team.
3. The number of goals a team scores is termed as "Goals For" and the number of goals it concedes is
termed as "Goals Against". We get the "Goal Difference" by subtracting "Goals Against" from "Goals For".
The ranking of the teams is decided on the total points. If two teams are tied on their total points, the team
which has a higher Goal Difference gets the higher rank. If the tie cannot be resolved on Goal Difference,
Goals For is checked followed by Goals Against. If the tie persists, the teams are ranked in the ascending
order of their names.
Table 1 provides data on the current top 13 teams based on the overall situation, i.e., by taking into
account both home matches and away matches of each team.
Table 2 provides data on the current top 13 teams based on home matches only.
Chart 1 provides a plot of the goal difference of each of the 13 teams based on the overall situation.

Team
CH
SW
WB
MC
MU
WH
EV
AS
WG

OVERALL
M
W
3
3
3
2
3
2
3
2
3
2
3
2
3
2
3
1
3
1

D
0
1
1
1
0
0
0
2
1

GF
8
10
6
8
6
4
4
2
4

Team
WB
WH
CH
MC
SW
NC
FU
EV
MU

M
2
2
2
2
2
2
1
1
1

HOME
W
D
2
0
2
0
2
0
2
0
1
1
1
1
1
0
1
0
1
0

GF
5
4
6
6
5
3
5
1
3

GA
0
0
2
3
2
2
0
0
2

` 7
NC
FU
ST
SU

27.

3
7
3
2

TOT
NW
AS
ST

2
1
1
1

0
0
0
0

2
1
1
1

2
1
0
0

2
1
0
0

2. CH

3. WG

4. SW

5. WB

2. 3

3. 4

4. 5

5. 6

Let us define a term pos as the difference between "home rank" and "away rank". Which of the following
has the maximum value of pos(Considering only the teams for which the information is given)??
1. AS

30.

1
0
3
2

Considering away matches only, the least number of teams with either 0 or 1 point is:
1. 2

29.

1
1
0
0

Considering away matches only, which of the following teams is the second ranking team (Considering
only the teams for which the information is given)?
1. AS

28.

3
3
3
2

2. WB

3. WH

4. MC

5. SW

How many unique values of goal difference are there for away matches(Considering only the teams for
which the information is given)??
1. 5

2. 6

3. 6

4. 6 and 13

5. 6 and 14

` 8
31.

A medical practitioner has created different potencies of a commonly used medicine by dissolving tablets
in water and using the resultant solution.
Potency 1 solution: When I tablet is dissolved in 50 ml, the entire 50 ml is equivalent to one dose.
Potency 2 solution: When 2 tablets are dissolved in 50 ml, the entire 50 ml of this solution is equivalent to
2 doses,...and so on.
This way he can give fractions of tablets based on the intensity of infection and the age of the patient.
For a particular patient, he administers 10 ml of potency 1, 15 ml of potency 2 and 30 ml of potency 4.
The dosage administered to the patient is equivalent to
1. > 2 and < 3 tablets
3. > 3.25 and < 3.5 tablets

32.

2. > 3 and < 3.25 tablets


4. > 3.5 and < 3.75 tablets

5. > 3.75 and < 4 tablets

A computer program was tested 300 times before its release. The testing was done in three stages of 100
tests each. The software failed 15 times in Stage I, 12 times in Stage II, 8 times in Stage III,6 times in
both Stage I and Stage II, 7 times in both Stage II and Stage III, 4 times in both Stage I and Stage III, and
4 times in all the three stages. How many times the software failed in a single stage only?
1. 10

2. 13

3. 15

4. 17

5. 35

DIRECTIONS for questions 33 to 35: Answer these questions which are based on the following information.
The following graph shows the revenue (in $ million) of three companies in their initial six years of operations, in
an economy which is characterised by a persistent inflation.

33.

In 2010, which could be a valid statement about the revenues (adjusted for inflation) of these three
companies?
1. Revenues of all three companies were equal.
2. Revenues of all three companies could be equal.
3. Revenue of Yahoo was definitely less than Facebook which was definitely less than Google.
4. Total of Yahoo and Facebook was definitely higher than than of Google.
5. Data insufficient.

` 9
34.

The difference in the average percentage increase in revenues, from 4th to 6th year, of Yahoo and
Facebook is:
1. 35%

35.

2. 1700

5. 45%

3.1900

4. 2100

5. None of the above

2. February 7, 2012
4. February 9, 2012

5. None of the above.

Ramesh analysed the monthly salary figures of five vice presidents of his company. All the salary
figures are integers. The mean and the median salary figures are Rs 5 lakh, and the only mode is Rs 8
lakh. Which of the options below is the sum (in Rs lakh) of the highest and the lowest salaries?
1. 9

38.

4. 50%

Three Vice Presidents (VP) regularly visit the plant on different days. Due to labour unrest, VP
(HR) regularly visits the plant after a gap of 2 days. VP (Operations) regularly visits the plant after a gap
of 3 days. VP (Sales) regularly visits the plant after a gap of 5 days. The VPs do not deviate from their
individual schedules. CEO of the company meets the VPs when all the three VPs come to the plant
together. CEO is one leave from January 5th to January 28th, 2012. Last time CEO met the VPs on
January 3, 2012. When is the next time CEO will meet all the VPs?
1. February 6,2012
3. February 8, 2012

37.

3. 55%

What would have been Facebook's revenue (in $ million) in its sixth year of operation if the
company had matched Google's percentage growth in revenues from the fifth to the sixth year?
Choose the option that is nearest to the answer.
1.1625

36.

2. 40%

2.10

3. 11

4. 12

5. None of these

Lionel and Ronaldo had a discussion on the ages of Jose's sons. Ronaldo made following statements
about Jose's sons:
i) Jose has three sons.
ii) The sum of the ages of joses sons is 13.
iii) The product of the ages of the sons is same as the age of Lionel.
iv) Joses eldest son, Zizou weights 32 kilos.
v) The sum of the ages of the younger sons of Jose 4.
vi) Jose has fathered a twin.
vii) Jose is not the father of a triplet.
viii) The LCM of the ages of Joses is more than the sum of their ages.
Which of the following combination gives information sufficient to determine the ages of Jose's sons?
1. i) ii), iii) and iv)

39.

3. i), ii), iii) and v)

4. i), ii), v) and vii)

5. i), ii), v) and vi)

Gopal sells fruit juice mixture using orange juice and pineapple juice. Gopal prepares this mixture
by drawing out a jug of orange juice from a 10 litre container filled with orange juice, and replacing it with
pineapple juice. If Gopal draws out another jug of the resultant mixture and replaces it with pineapple
juice, the container will have equal volumes of orange juice and pineapple juice. The volume of the jug, in
litres, is
1. 2

40.

2. i), ii), iv) and vi)

2. > 2 and < 2.5

3. 2.5

4. > 2.5 and < 3

5. >3

The sides of a rhombus ABCD measure 2 cm each and the difference between two angles is 90 then the
area of the rhombus is
1.

2 sq. cm

2. 2 2 sq. cm

3. 3 2 sq. cm

4. 4 2 sq. cm

` 10
41.

Suresh, who runs a bakery, uses a conical shaped equipment to write decorative labels (e.g., Happy
Birthday etc.) using cream. The height of this equipment is 7 cm and the diameter of the base is 5 mm. A
full charge of the equipment will write 330 words on an average. How many words can be written using
three fifth of a litre of cream?
1. 450900

42.

3. Gina

4. Lina

5. Bina

2. 2 : 3

3. 1 : 4

4. 1 : 5

2. 4125.00

3. 4155.00

4. 5100.00

5. 5355.00

2. 100 times

3. 10 times

4. No change

5. None of these

A potter asked his two sons to sell some pots in the market. The amount received for each pot was
same as the number of pots sold. The two brothers spent the entire amount on some packets of
potato chips and one packet of banana chips. One brother had the packet of banana chips along with
some packets of potato chips, while the other brother just had potato chips. Each packet of potato chips
costs Rs. 10/- and the packet of banana chips costs less than Rs. 10/-. The packets of chips were divided
between the two brothers so each brother received equal number of packets. How much money should
one brother give to the other to make the division financially equitable?
1. 1

47.

2. Mina

A spherical metal of radius 10 cm is molten and made into 1000 smaller spheres of equal sizes. In
this process the surface area of the metal is increased by:
1. 1000 times

46.

5. None of these

A man borrows Rs. 6000 at 5% interest, on reducing balance, at the start of the year. If he repays Rs.
1200 at the end of each year, find the amount of loan outstanding, in Rs., at the beginning of the third
1. 3162.75

45.

4. 432000

If Sn denotes the sum of the first n terms in an Arithmetic Progression and S1:S4 = 1 : 10 then the ratio of
the first term to fourth term is
1. 1 : 3

44.

3. 460000

Tina, Mina, Gina, Lina, and Bina are 5 sisters, aged in that order, with Tina being the eldest. Each of them
had to carry a bucket of water from a well to their house. Their buckets' capacities were
proportional to their ages. While returning, equal amount of water got splashed out of their buckets. Who
lost maximum amount of water as a percentage of the bucket capacity?
1. Tina

43.

2. 451000

2. 2

3. 4

4. 5

5. 7

Nikhil's mother asks him to buy 100 pieces of sweets worth Rs. 100/-. The sweet shop has 3 kinds of
sweets, kajubarfi, gulabjamun and sandesh. Kajubarfi costs Rs. 10/- per piece, gulabjamun costs Rs. 3/per piece and sandesh costs 50 paise per piece. If Nikhil decides to buy at least one sweet of each type,
how many gulabjamuns should he buy?
1. 1

2. 2

3. 3

4. 4

5. 5

DIRECTIONS for questions 48 & 49: based on the following information.


Ramya, based in Shanpur, took her car for a 400 km trip to Rampur. She maintained a log of the odometer
readings and the amount of petrol she purchased at different petrol pumps at different prices (given below). Her
car already had 10 litres of petrol at the start of the journey, and she first purchased petrol at the start of the
journey, as given in table below, and she had 5 litres remaining at the end of the journey.

` 11
Start of Journey
End of Journey
48.

4. 9.50

5. None of these

2. 1724

3. 1734

4. 1744

5. Data insufficient

2. 4800 m

3. 5600 m

4. 6400 m

5. 7200 m

2. 9

3. 10

4. 12

5. 15

City Bus Corporation runs two buses from terminus A to terminus B, each bus making 5 round trips in a
day. There are no stops in between. These buses ply back and forth on the same route at different but
uniform speeds. Each morning the buses start at 7 AM from the respective terminuses.
They meet for the first time at a distance of 7 km from terminus A. Their next meeting is at a
distance of 4 km from terminus B, while travelling in opposite directions. Assuming that the time
taken by the buses at the terminuses is negligibly small, and the cost of running a bus is Rs. 20 per km,
find the daily cost of running the buses (in Rs.).
1. 3200

53.

3. 9.00

Ram prepares solutions of alcohol in water according to customers' needs. This morning Ram has
prepared 27 litres of a 12% alcohol solution and kept it ready in a 27 litre delivery container to be
shipped to the customer. Just before delivery, he finds out that the customer had asked for 27 litres of
21% alcohol solution. To prepare what the customer wants, Ram replaces a portion of 12% solution by
39% solution. How many litres of 12% solution are replaced?
1. 5

52.

2. 8.50

A city has a park shaped as a right angled triangle. The length of the longest side of this park is 80
m. The Mayor of the city wants to construct three paths from the corner point opposite to the longest side
such that these three paths divide the longest side into four equal segments. Determine the sum of the
squares of the lengths of the three paths.
1. 4000 m

51.

Rate of Petrol (Rs./litre)


30
35
40

Her car's tank-capacity is 35 litres. Petrol costs Rs. 45/ litre in Rampur. What is the minimum amount of
money she would need for purchasing petrol for the return trip from Rampur to Shanpur,using the same
route? Assume that the mileage of the car remains unchanged throughout the route, and she did not use
her car to travel around in Rampur.
1. 1714

50.

Petrol purchased (Litre)


20
15
10

What has been the mileage (in kilometres per litre) of her car over the entire trip?
1. 8.00

49.

Odometer Reading (Km)


400
600
650
800

2. 4000

3. 6400

4. 6800

5. None of these

Shyam, a fertilizer salesman, sells directly to farmers. He visits two villages A and B. Shyam starts from
A, and travels 50 metres to the East, then 50 metres North-East at exactly 45 to his earlier direction, and
then another 50 metres East to reach village B. If the shortest distance between villages A and B is in the
form of a b c metres, find the value of a+b+c.
1. 52

54.

2. 54

3. 58

4. 59

5. None of these

Three truck drivers, Amar, Akbar and Anthony stop at a road side eating joint. Amar orders 10 rotis, 4
plates of tadka, and a cup of tea. Akbar orders 7 rotis, 3 plates of tadka, and a cup of tea. Amar pays Rs.
80 for the meal and Akbar pays Rs. 60. Meanwhile, Anthony orders 5 rotis, 5 plates of tadka and 5 cups
of tea. How much (in Rs.) will Anthony pay?
1. 75

2. 80

3. 95

4. 100

5. None of these

` 12
55.

A property dealer bought a rectangular piece of land at Rs. 1000/sq ft. The length of the plot is less than
twice its breadth. Due to its size, there were no buyers for the full plot. Hence he decided to sell it in
smaller sized pieces as given below.
The largest square from one end was sold at Rs. 1200/sq.ft. From the remaining rectangle the largest
square was sold at Rs. 1150/sq.ft. Due to crash in the property prices, the dealer found it difficult to make
profit from the sale of the remaining part of the land. If the ratio of the perimeter of the remaining land to
the perimeter of the original land is 3:8, at what price (in Rs.) the remaining part of the land is to be sold
such that the dealer makes an overall profit of 10% ?
1. 500/sqft

2. 550/sqft

3. 600/sqft

4. 650/sqft

5. None of these

DIRECTIONS for questions 56 & 57: The following pie chart shows the percentage distribution of runs scored by
a batsman in a test innings.

56.

If the batsman has scored a total of 306 runs, how many 4s and 6s did he hit?
1. 3 and 33 respectively
3. 3 and 22 respectively
5. 5 and 22 respectively

57.

The batsman has scored a total of 306 runs. If 5 of the dot balls had been hit for 4s, and if two of the
shots for which the batsman scored 3 runs each had fetched him one run instead, what would have been
the central angle of the sector corresponding to the percentage of runs scored in 4s?
1. 40.790

58.

2. 4 and 23 respectively
4. 33 and 1 respectively

2. 111.570

3. 35.780

4. 100.520

5. 70.640

Ram and Shyam form a partnership (with Shyam as working partner) and start a business by
investing Rs. 4000 and Rs. 6000 respectively. The conditions of partnership were as follows:
In case of profits till Rs. 200,000 per annum, profits would be shared in the ratio of the invested capital.
Profits from Rs. 200,001 till Rs. 400,000 Shyam would take 20% out of the profit, before the division of
remaining profits, which will then be based on ratio of invested capital.
Profits in excess of Rs. 400,000, Shyam would take 35% out of the profits beyond Rs. 400,000, before
the division of remaining profits, which will then be based on ratio of invested capital.

` 13
If Shyam's share in a particular year was Rs. 367000, which option indicates the total business profit (in
Rs.) for that year?
1. 5,20,000

2. 5,30,000

3. 5,40,000

4. 5,50,000

5. None of these

DIRECTIONS for questions 59 to 62: Answer based on the following information.


The following graphs give annual data of Assets, Sales (as percentage of Assets) and Spending on Corporate
Social Responsibility (CSR) (as percentage of Sales), of a company for the period 2004 - 2009.

59.

In which year was the increase in spending on CSR, vis--vis the previous year, the maximum?
1. 2005

2. 2006

3. 2007

4. 2008

5. 2009

` 14
60.

Of the years indicated below, in which year was the ratio of CSR/ Assets the maximum?
1. 2004

61.

3.2006

4.2007

5. 2008

What was the maximum value of spending on CSR activities in the period 2004-2009 ?
1. Rs. 0.5 lakh

62.

2. 2005

2. Rs. 1.0 lakh

3. Rs. 2.0 lakh

4. Rs. 3.0 lakh

5. Rs. 4.0 lakh

In which year, did the spending on CSR (measured in Rs.) decline, as compared to previous year?
1. 2006

2. 2007

3. 2008

4. 2009

5. None of the above

DIRECTIONS for questions 63 to 65: Answer based on the following information.


Five years ago Maxam Glass Co. had estimated its staff requirements in the five levels in their organization as:
Level-1: 55; Level-2: 65; Level-3: 225; Level-4: 255 & Level-5: 300. Over the years the company had recruited
people based on ad-hoc requirements, in the process also selecting ex-defence service men and ex-policemen.
The following graph shows actual staff strength at various levels as on date.

63.

The level in which the Ex-Defence Servicemen are highest in percentage terms is:
1. 1

64.

3. 3

4. 4

5. 5

If the company decides to abolish all vacant posts at all levels, which level would incur the highest
reduction in percentage terms ?
1. 1

65.

2. 2

2. 2

3. 3

4. 4

5. 5

Among all levels, which level has the lowest representation of Ex-policemen ?
1. 1

2. 2

3. 3

4. 4

5. 5

` 15
66.

In a locality, there are ten houses in a row. On a particular night a thief planned to steal from three houses
of the locality. In how many ways can he plan such that no two of them are next to each other?
1. 56

67.

2. 73

3. 80

4. 120

5. None of the above

Let an = 11111111, where 1 occurs n number of times. Then,


i. a741 is not a prime.
ii. a534 is not a prime.
iii. a123 is not a prime.
iv. a77 is not a prime.
1. (i) is correct.
3. (ii) and (iii) are correct
5. None of them is correct.

2. (i) and (ii) are correct.


4. All of them are correct.

DIRECTIONS for questions 68 & 69: Answer based on the following information.
Income-Tax Rates for Financial Year 2009-10
Individual & HUF below age of 65 years
Income up to Rs. 1,60,000/Rs. 1,60,001 to Rs. 3,00,000/Rs. 3,00,001 to Rs. 5, 00,000/Above Rs. 5,00,001

Women below age of 65 years


Income up to Rs. 1,90,000/Rs. 1,90,001 to Rs. 3,00,000/Rs. 3,00,001 to Rs. 5,00,000/Above Rs. 5,00,001

Tax Rates
Nil
10%
20%
30%

Total income tax payable is obtained by adding two additional surcharges on calculated income tax.
Education Cess: An additional surcharge called 'Education Cess' is levied at the rate of 2% on the amount of
income tax.
Secondary and Higher Education Cess: An additional surcharge called 'Secondary and Higher Education Cess' is
levied at the rate of 1% on the amount of income tax.
68.

Sangeeta is a young working lady. Towards the end of the financial year 2009-10, she found her total
annual income to be Rs. 3,37,425/-. What % of her income is payable as income tax?
1. 5.64

69.

4. 8.10

5. None of the above

2. 14,45,522/-

3. 14,47,169/-

4. 15,70,718/-

5. None of the above

What is the maximum possible value of (21 Sin X + 72 Cos X)?


1. 21

71.

3. 7.38

Mr. Madan observed his tax deduction at source, done by his employer, as Rs. 317,910/-. What was his
total income (in Rs.) if he neither has to pay any additional tax, nor is eligible for any refund?
1. 13,48,835/-

70.

2. 6.82

2. 57

3. 63

4. 75

5. None of the above

The scheduling officer for a local police department is trying to schedule additional patrol units in each of
two neighbourhoods - southern and northern. She knows that on any given day, the probabilities of major
crimes and minor crimes being committed in the northern neighbourhood were 0.418 and 0.612,
respectively, and that the corresponding probabilities in the southern neighbourhood were 0.355 and
0.520. Assuming that all crime occur independent of each other and likewise that crime in the two
neighbourhoods are independent of each other, what is the probability that no crime of either type is
committed in either neighbourhood on any given day?
1. 0.069

2. 0.225

3. 0.690

4. 0.775

5. None of the above

` 16
DIRECTIONS for questions 72 & 73: Answer based on the following information.
A man standing on a boat south of a light house observes his shadow to be 24 meters long, as measured at the
sea level. On sailing 300 meters eastwards, he finds his shadow as 30 meters long, measured in a similar
manner. The height of the man is 6 meters above sea level.
72.

The height of the light house above the sea level is:
1. 90 meters

73.

2. 400 meters

3. 500 meters

4. 600 meters

5. None of the above

2. 5

3. 8

4. 6

5. None of the above.

2. 1/6

3. 1/4

4. 1/3

5. 1

In a city, there is a circular park. There are four points of entry into the park, namely - P, Q, R and S.
Three paths are constructed which connected the points PQ, RS, and PS. The length of the path PQ is 10
units, and the length of the path RS is 7 units. Later, the municipal corporation extended the paths PQ
and RS past Q and R respectively, and they meet at a point T on the main road outside the park. The
path from Q to T measures 8 units, and it was found that the angle PTS is 60. Find the area (in square
units) enclosed by the paths PT, TS, and PS.
1. 36 3

77.

5. 106 meters

There are four machines in a factory. At exactly 8 pm, when the mechanic is about to leave the factory,
he is informed that two of the four machines are not working properly. The mechanic is in a hurry, and
decides that he will identify the two faulty machines before going home, and repair them next morning. It
takes him twenty minutes to walk to the bus stop. The last bus leaves at 8:32 pm. If it takes six minutes to
identify whether a machine is defective or not, and if he decides to check the machines at random, what
is the probability that the mechanic will be able to catch the last bus?
1. 0

76.

4. 100 meters

A 25 ft long ladder is placed against the wall with its base 7 ft from the wall. The base of the ladder is
drawn out so that the top comes down by half the distance that the base is drawn out. This distance is:
1. 7

75.

3. 96 meters

What is the horizontal distance of the man from the light house in the second position?
1. 300 meters

74.

2. 94 meters

2. 54 3

3. 72 3

4. 90 3

5. None of the above

There are 240 second year students in a B-School. The Finance area offers 3 electives in the second
year. These are Financial Derivatives, Behavioural Finance, and Security Analysis. Four students have
taken all the three electives, and 48 students have taken Financial Derivatives. There are twice as many
students who study Financial Derivatives and Security Analysis but not Behavioural Finance, as those
who study both Financial Derivatives and Behavioural Finance but not Security Analysis, and 4 times as
many who study all the three. 124 students study Security Analysis. There are 59 students who could not
muster courage to take up any of these subjects. The group of students who study both Financial
Derivatives and Security Analysis but not Behavioural Finance, is exactly the same as the group made up
of students who study both Behavioural Finance and Security Analysis. How many students study
Behavioural Finance only?
1. 29
4. 35

2. 30
5. None of the above options

3. 32

` 17
DIRECTIONS for question: Question is followed by two statements labeled as I and II. Decide if these
statements are sufficient to conclusively answer the question. Choose the appropriate answer from the options
given below:
1. Statement I alone is sufficient to answer the question.
2. Statement II alone is sufficient to answer the question.
3. Statement I and Statement II together are sufficient, but neither of the two alone is sufficient to answer the
question.
4. Either Statement 1 or Statement II alone is sufficient to answer the question.
5. Neither Statement I nor Statement II is necessary to answer the question.
78.

Given below is an equation where the letters represent digits.


(PQ) . (RQ) = XXX. Determine the sum of P + Q + R + X.
I. X = 9.
II. The digits are unique.

DIRECTIONS for questions 79 & 80: Read the information given below and answer the question.
A truck travelled from town A to town B over several days. During the first day, it covered 1/p of the total distance,
where p is a natural number. During the second day, it travelled 1/q of the remaining distance, where q is a
natural number. During the third day, it travelled 1/p of the distance remaining after the second day, and during
the fourth day, 1/q of the distance remaining after third day. By the end of the fourth day the truck had travelled
3/4 of the distance between A and B.
79.

The value of p + q is
1. 4

80.

2. 30

For how many integers n,


1. 0

82.

3. 6

4. 7

5. 8

If the total distance is 100 kilometres, the minimum distance that can be covered on day 1 is _________
kilometers.
1. 25

81.

2. 5

2. 1

3. 33

4. 35

5. 40

n
is the square of an integer?
20 n
3. 2

4. 3

5. 4

Eight cities A, B, C, D, E, F, G and H are connected with one-way roads R1, R2, R3, R4, R5 and R6 in the
following manner:
R1 leads from A to C via B;
R2 leads from C to D and then via B to F;
R3 leads from D to A and then via E to H;
R4 leads from F to B via G;
R5 leads from G to D; and
R6 leads from F to H.
The minimum number of road segments that have to be blocked in order to make all traffic from B to D
impossible is
1. 5

2. 4

3. 3

4. 2

5. 1

` 18
83.

We define a function f on the integers f(x) = x/10, if x is divisible by 10, and f(x) = x + 1 if x is not divisible
by 10. If A0 = 1994 and An+1 = f(An). What is the smallest n such that An = 2?
1. 9

84.

87.

4. 2

2. 3

3. 4

4. 5

4. 2 7

5. Not uniquely determined.

5. 3

5. None of the above.

3.

A brokerage house offers 3 stock portfolios. Portfolio I consists of 2 blocks of common stock and 1
municipal bond. Portfolio II consists of 4 blocks of common stock. 2 municipal bonds and 3 blocks of
preferred stock. Portfolio III consists of 2 blocks of common stock. 2 municipal bond and 3 blocks of
preferred stock. A customer wants 12 blocks of common stock. 6 municipal bonds and 6 preferred stocks.
How many portfolio III should be offered?
2. 2

3. 3

4. 4

5. None of the above

Triangle ABC has vertices A(0, 0), B(0, 6) and C(9, 0). The points P and Q lie on side AC such that AP =
PQ = QC. Similarly, the points R and S lie on side AB such that AR = RS = SB. If the line segments PB
and RC intersect at X. then the slope of the line AX is
2. -2/3

3. 3/2

4. -3/2

5. None of the above.

In a rainy season, Jamshedpur experienced some rain for thirteen days. A morning rain was always
followed by a clear evening. An evening rain was always preceded by a clear morning. A total of 9
mornings and 12 evenings were clear. How many rainless days did Jamshedpur have in the period?
2. 6

3. 5

4. 4

5. 3

a, b, c, d and e be non-negative real numbers such that a + b + c + d + e = 10. Let X be the maximum of
the numbers a + b, b + c, c + d and d + e. The least possible value of X lies in the interval
1. [0, 2]

91.

3. 1

2. 1 5

1. 7
90.

2. 0

1. 1 5

1. 2/3
89.

5. An never equals 2

ABCD is a rectangle. The points P and Q lie on AD and AB respectively. If the triangles PAQ, QBC and
PCD all have the same areas and BQ = 2 then AQ =

1. 1
88.

4. 1993

In a certain factory, each day the expected-number of accidents is related to the number of overtime
hours by a linear equation. Suppose that on one day there were 1000 overtime hours logged and 8
accidents reported, and on another day there were 400 overtime hours logged and 5 accidents. What are
the expected numbers of accidents when no overtime hours are logged?
1. 2

86.

3. 128

If the polynomial x3 + px + q has three distinct roots, then which of the following is a possible value of p?
1. -1

85.

2. 18

2. [2, 3]

3. [3, 4]

4. [4, 10]

5. [7, 10]

A management institute has six senior professors and four junior professors. Three professors are
selected at random for a Government project. The probability that at least one of the junior professors
would get selected is:
1. 5/6

2. 2/3

3. 1/5

4. 1/6

5. None of the above

` 19
DIRECTIONS for questions 92 to 94: Questions are followed by two statements labelled as (A) and (B). You
have to decide if these statements are sufficient to conclusively answer the question. Choose
A. If Statement (A) alone is sufficient to answer the question.
B. If Statement (B) alone is sufficient to answer the question
C. If Statement (A) and Statement (B) together are sufficient but neither of the two alone is sufficient to answer
the question
D. If either Statement (A) or Statement (B) alone is sufficient to answer the question.
E. Both Statement (A) and Statement (B) are insufficient to answer the question.
92.

What is the maximum value of a/b?


A. a, a + b and a + 2b are three sides of a triangle.
B. a and b both are positive.

93.

ABC is a triangle with B = 90. What is the length of the side AC?
A. D is the midpoint of BC and E is the midpoint of AB.
B. AD = 7and CE = 5

94.

Five integers A, B, C, D and E are arranged in such a way that there are two integers between B and C
and B is not the greatest. There exists one integer between D and E and D is smaller than E. A is not the
smallest integer. Which one is the smallest?
A. E is the greatest
B. There exists no integer between B and E.

95.

For which value of non-negative 'a' will the system x2 y2 = 0, (x - a)2 + y2 = 1 have exactly three real
solutions?
A. 2

96.

2. 1

3.

4. 2

5. No such 'a' exists.

Let {An} be a unique sequence of positive integers satisfying the following properties:
A1 = 1, A2 = 2, A4 = 12, and An + 1.An-1 = An2 1 for n = 2, 3, 4 ,
Then, A7 is
1. 60

2. 120

3. 149

4. 169

5. 187

DIRECTIONS for questions 97 to 100: Read the data below and choose the correct option for the questions that
follow.
Queen Airlines offers the following Privilege programme: There are 5 membership tiers, each with its own set of
enhanced tier-specific benefits. New members join at the Blue tier level, then upgrade to the Blue Plus, with the
added benefit of Tele check-in. The programme also has three elite tiers - Silver, Gold and Platinum. The 6-month
fast-track upgrade and the 12-month standard upgrade system operate in parallel -implying that whenever a
particular passenger satisfies conditions, either in terms of number of flights in the stipulated period or in terms of
accumulated Queen-Miles in either of the two systems, they are automatically upgraded to the next tier. The 6month upgrade system considers the data in rolling 6 months period. The same holds for the 12-month period. If
conditions for more than one upgrade are satisfied, the passenger is given the higher of the two.

` 20

Tier Upgrade
Blue to Blue Plus
Blue Plus to Silver
Silver to Gold
Gold to Platinum

Cumulative Queen Airways


flights/Cumulative Status Queen-Miles
6-month Fast Track
12-month Standard
Upgrade System
Upgrade System
3 / 3,000
Not Applicable
Not Applicable
5/12,500
5/12,500
10/20,000
10/20,000
20/30,000

Benefits and Privileges


Blue
Tele Check-in
Additional baggage allowance
Confirmed upgrade vouchers
Guaranteed reservations up to 24 hours prior to
departure
Cancellation fees waived on published fares

Blue
Plus

Silver

Gold

10 kgs
1

20 kgs
3

Platinu
m

35 kgs
5

Distances between cities in Queen-miles


Kolkata
Mumbai
Hyderabad
Bangalore
Coimbatore
Guwahati
Chennai

1461
1407
1499
2061
2401
1959
2095
Delhi

1987
1516
1881
2167
1081
1676
Kkata

711
998
1265
2746
1329
Mum

562
902
2370
688
Hbad

340
2932
331
Blore

3209
491
Ctore

2718
Ghy

Mr. Kakkar, a newly recruited MBA from a business school, started his career with the start of the year 2007. His
travel plans for the year of 2007 is given below in the table.
Date
2.1.2007
8.1.2007
3.2.2007
8.3.2007
20.3.2007
11.4.2007
30.4.2007
4.7.2007
20.7.2007
2.9.2007
11.9.2007
22.9.2007
1.10.2007
11.10.2007
4.11.207

From
Mumbai
Hyderabad
Mumbai
Delhi
Guwahati
Kolkata
Guwahati
Chennai
Guwahati
Kolkata
Hyderabad
Guwahati
Chennai
Guwahati
Chennai

To
Hyderabad
Mumbai
Delhi
Guwahati
Kolkata
Guwahati
Chennai
Guwahati
Kolkata
Hyderabad
Guwahati
Delhi
Bangalore
Chennai
Delhi

` 21
29.11.2007
1.12.2007
31.12.2007
97.

2. February

2. 8

4. April

5. May

3. 9

4. 10

5. 11

The difference in complete calendar months, discounting any partial months, between the first tier
upgrade and the last tier upgrade for Mr. Kakkar is:
1. 7

100.

3. March

The number of complete calendar months, discounting any partial months, for which Mr. Kakkar avails the
gold tier membership is:
1. 7

99.

Hyderabad
Guwahati
Mumbai

In which month will Mr. Kakkar become eligible for guaranteed reservations up to 24 hours prior to
departure?
1. January

98.

Delhi
Hyderabad
Guwahati

2. 8

3. 9

4. 10

5. 11

If 6-month upgrade had not been in operation, then Mr. Kakkar would have reached Gold tier in the month
of
1. June

2. July

3. August

4. September

5. October

DIRECTIONS for questions 101 and 102: The graph given below contains data pertaining to number of
electronic commerce transactions that have taken place in the last six months of the financial year 2005. This
graph contains data related to private consumption and does not include corporate electronic commerce activities.
Numbers mentioned above the bar graphs are in millions and average price per unit is mentioned in the brackets.

Number of Transactions

Online Consumer Trends


Books ($5)

Video/DVDs/Games ($10)

Airline Tickets/Reservation ($20)

Clothing/Accessories/Shoes ($15)

50
40

44
37

37
32

30
20

25
17

20

24

31

28
21

23

22

22
16

25

21
16
9

10

0
Asia Pacific

Europe

North
America
Regions

Latin America South Africa

` 22
101.

For which product category is the revenue contribution of Asia Pacific region the maximum?
1. Books
2. Video DVDs / Games
3. Both Airline Tickets Reservation and Clothing / Accessories Shoes
4. Clothing Accessories / Shoes
5. Airline Tickets / Reservation

102.

If the airline ticket purchases made through internet increase by 20% and the average price of the airline
ticket increases by 25% then the net increase in revenues from the ecommerce activities worldwide will
be _________ percent of the corresponding pre-price-increase revenues.
1. 18

2. 19

3. 20

4. 21

5. 22

DIRECTIONS for questions 103 and 104: Study the aggregate financial ratios of all registered Indian
manufacturing companies in the table below to answer the questions that follow.
All figures are as % of net sales unless otherwise mentioned
2000 2001 2002 2003 2004
PBDIT
13.1
11.7
12.3
13.3
14.4
PBDT
8.1
7.1
8
9.9
11.8
PBIT
9.4
8.4
8.7
9.9
11
PAT
3.2
2.8
2.7
4.4
6
Raw Material expense
41
40.6
43.1
45.5
45.7
Salaries and wages
5.9
5.7
5.6
5.3
4.9
Interest payments
4.6
4.3
4
3.1
2.3
Operating profit
5.2
4.2
4.9
6.7
8
Net sales (% Growth Over Previous
18.4
19.3
2.6
15.7
15.2
Year)
103.

19.9

What is the annual growth rate in aggregate PAT of the Indian manufacturing companies in the financial
year 2005 as compared to that in the financial year 2004?
1. 15.0 percent

104.

2005
14.7
12.7
11.6
6.9
47.1
4.4
1.7
8.7

2. 5.7 percent

3. 88.6 percent

4. 37.8 percent

5. None of the above.

In which year the annual growth rate in the aggregate Salaries and Wages expense was maximum?
1. 2005

2. 2004

3. 2003

4. 2002

5. 2001

DIRECTIONS for questions 105 & 106: In the magic square below, the sum of the numbers in each of the rows,
in each of the diagonals and in each of the columns is equal.
y
y1

105.

x
y+1

y2

If x = 10, then the possible value of y is


1. 5

2. 7

3. 9

4. None of the above

` 23
106.

If the sum of the numbers in each rows, in each of the diagonals, and in each of the columns is 21, a
possible value of x is
1. 5

107.

3. 9

4. None of the above

If the two circles x2 + y2 - 2x - 6y = r2 - 10 and x2 + y2 - 8x + 2y + 8 = 0 intersect at two points then


1. 0 < r < 2

108.

2. 7

2. 2 < r < 8

3. 8 < r < 10

4. None of the above

How many natural numbers leave a remainder of 41 with the dividend being 1997?
1. 6

2. 11

3. 27

4. None of the above


2

109.

c c
is
a b

If a, b and c are positive real numbers with a2 + b2 = c2, then the minimum value of
1. 4

110.

2. 3. a+1C4

3. 3. b+1C2

4. 3. a+1C2

2. 20% decrease

3. 20% increase

4. 25% increase

Consider the sequence x1 = 5, x2 = 25, xn+2 = gcd(xn+1 , xn ) + xn, for n 1, where gcd(a, b) means greatest
common devisor of a and b, then the least common multiple of x19 and x20 is:
1. 2440

113.

4. 64

The length and the breadth of a cuboid are increased by 25% and 33 1/3% respectively. What is the
percentage change in the height of the cuboid such that the volume, which was 600 cu. cm, becomes 800
cu. cm?
1. 25% decrease

112.

3. 16

If aC2 = b, then bC2 =


1. 3. b+1C4

111.

2. 8

2. 2560

3. 4800

4. 5120

A real function 'F' defined on all pairs of non-negative integers is given. This function satisfies the
following conditions:
F(0, 0) = 0,
F(2x, 2y) = F(2x+1, 2y+l) = F(x, y)
F(2x, 2y + l) = F(2x + 1, 2y) = F(x, y) + 1
The values of F(11, 7) is
1. 2

114.

2. 3

3. 4

4. 6

An urn contains 6 red balls and 4 black balls. Three random drawings are made, such that the ball drawn
is always replaced, and, in addition, 2 balls of the colour drawn are added to the urn. The probability of a
draw of three black balls is
1. 53/70

2. 2/7

3. 6/35

4. None of the above

` 24
115.

A firm's short-run production function is given by P = L2(30 L)(1 + 2L)2, where L is the-number of
persons-employed. What level of employment gives the maximum output?
1. -6

2. 6

3. 5

4. 4

Directions for questions 116 and 117: Read the information carefully and answer the questions.
116.

A batch contains 20 electronic components, 5 of which are defective. From this batch, 2 components are
selected in succession. Let the event A = the first component selected is defective, and event B = the
second component selected is defective.
Which of the following is true?
1. A and B are independent
2. A and B are dependent
3. A and B are mutually exclusive and collectively exhaustive.
4. A and B are mutually exclusive and equally likely.

117.

Which of the following is true?


1. P(A) < P(B)

118.

2. Rs. 6120

2. b2 = 4a3

2. 0

3 ) and ( 3 ,) only

3. (- 3 , 0) and ( 3 , ) only

3. 4a2 b2 = 0

4. None of the above

3.

1
2

4.

3
2

2. (-, - 3 ) and (0, 3 ) only


4. None of the above

The cross section of a circus tent is in the shape of an equilateral triangle surmounting a rectangle. If the
side of the triangle is 24 ft, and length of the prism is 30 ft and the height of the rectangular part is 15 ft,
then what is the total surface area of the tent?(in sq. ft)?
1. 1440 + 723

123.

4. Rs. 5740

The function f(x) = x4 - 6x2 is increasing in the interval


1. (0,

122.

3. Rs. 5920

Which of the following denotes the minimum value of | z | | z 1 | ?


1. 1

121.

4. P(B|A) = P(A|B)

If the equation x2 - 3ax + b = 0 has two equal roots, then which of the following must be true?
1. 4a2 + b2 = 0

120.

3. P(A) = P(B)

Mr. Basu is planning to buy a two wheeler. He has approached a public sector bank for a loan of Rs.
25000. The interest rate for the loan is 12% per annum compounded annually with the interest being
charged on the total amount. Mr. Basu wants to repay through monthly installments of Rs. 1000. The
amount of balance loan left to be repaid after 2 years is:
1. Rs. 7360

119.

2. P(A) > P(B)

2. 4550 + 2883

3. 1620 + 1443

4. 3060 + 2883

What is the total number of palindromes between 10 and 100000 (A palindromic number gives the same
number even on reversing the order of the digits e.g. 2345, when reversed it gives 5432)
1. 3429

2. 1294

3. 1089

4. 4069

` 25
124.

A swimming pool is rectangular in shape. Its length is 10 m. and width 6 m. The pool is 2.5m deep at the
deep end and 0.8m deep at the shallow end with a uniform slope between the deep and shallow ends. If
a tap provides water at a rate 90 litres/min., how long would it take to fill the pool. [1 m3= 1000 litres].
1. 18 hrs.

125.

2. 18.33 hrs.

3. 18.73 hrs.

a, b and c are three different digits from 1 to 9 forming the number abc (for example the digits 1, 2 and 3
would form the number 123). The smallest value of
1. 9.0

126.

4. None of the above.

2. 9.5

3. 10.0

abc
is
abc
4. 10.5

A malaria epidemic hits a large city. Each resident in the city is either sick, well, or a carrier. The
proportion of people in each of the categories is expressed by the matrix
AGE
15-35
0.1
0.2
0.1

Under 15
0.7
0.1
0.2

Well
Sick
Carrier

Over 35
0.6
0.3
0.1

The population of the city is distributed by age and sex as follows:


AGE

Under 15
15-35
Over 35

Male
150000
100000
400000

Female
200000
80000
420000

A health care agency wants to find out the number of female carriers. The number of female carriers is
1. 78,000
127.

2. 80,000

3. 90,000

4. 92,000

The real numbers x and y are such that x2 + xy + y2 = 1 and A = x3y + xy3
If x and y are positive real numbers, then the greatest possible value of A is
1. 0

2.

1
4

3. 2

4. None of the above

DIRECTIONS for questions 128 to 131: Questions are followed by two statements labeled as (A) and (B). You
have to decide if these statements are sufficient to conclusively answer the question. Give answer.
1.
2.
3.
4.
128.

If Statement (A) alone is sufficient to answer the question


If Statement (B) alone is sufficient to answer the question
If Statement (A) and Statement (B) together are sufficient but neither alone is sufficient to answer the
question
If neither Statement (A) nor Statement (B) is sufficient to answer the question.
Seven integers A, B, C, D, E, F and G are such that
No two integers are equal.
F is greater than G, but smaller than C.
Both A and B are greater than C.
D is greater than E.

` 26
Which is the smallest integer?
A. F is greater than E.
129.

AP and BQ are perpendicular to PQ. The line BP and AQ meet at S, and ST is perpendicular to PQ.
If AP = 6, and BQ = 4.5 then what is the length of ST?
A. PQ = 8

130.

B. PS = 5

ABCD is a parallelogram. E and F are the midpoints of BC and CD respectively. AE and AF intersect the
diagonal BD at P and Q. Diagonal AC intersects BD at R. What is the length of PQ?
A. BR = 6cm

131.

B. AD = 4 cm

f(x, y) is a polynomial in x and y. Is (x y) 2 x2y2 a factor of f(x, y)?


A. x y is a factor of f(x, y)

132.

B. E is greater than F.

B. f(x, y) is symmetric, i.e. f(x, y) = f(y, x).

A person starts from his house to office 25 min later than his usual time. So he increases the speed by
20% over his usual speed but still reaches his office 10 min late. Next day he starts 15 min later than his
usual time and goes at 1.5 kmph faster than his usual speed and reaches his office 3 min before the
office time. What is the distance between his house and office?
1. 8 km

2. 9 km

3. 12 km

4. 6 km

` 27

Analytical Reasoning
DIRECTIONS for questions 1 to 4: Read the following caselet and choose the best alternative for the questions
that follow.
The BIG and Colourful Company
You are running "BIG and Colourful (BnC)" company that sells books to customers through three retail formats:
1. You can buy books from bookstores,
2. You can buy books from supermarket,
3. You can order books over the Internet (Online).
Your manager has an interesting way of classifying expenses: some of the expenses are classified in terms of
size: Big, Small and Medium; and others are classified in terms of the colors, Red, Yellow, Green and Violet. The
company has a history of categorizing overall costs into initial cost* and additional costs. Additional costs are
equal to the sum of Big, Small and Medium expenses. There are two types of margins, contribution (sales minus
initial costs) and profit (contribution minus additional costs). Given below is the data about sales and costs of
BnC:
Sales

60000

Initial Costs

39000

Contribution (Sales - Initial Costs)

21000

Additional Costs
Big

9300

Small

3000

Medium

3500
15800

Profit (Contribution -Additional Cost)

5200

Each of the Big, Small and Medium cost is categorized by the manager into Red, Yellow, Green and Violet costs.
Breakdown of the additional costs under these headings is shown in the table below:
Expenses

Total

Red

Yellow

Green

Violet

Big
Small

9300
3000

5100

1200
400

1400
2000

1600
600

Medium

3500

400

1500

1400

200

Total

15800

5500

3100

4800

2400

Red, Yellow, Green and Violet costs are allocated to different retail formats. These costs are apportioned in the
ratio of number of units consumed by each retail format. The number of units consumed by each retail format is
given in the table below:

` 28

1.

Retail Format

Red

Yellow

Green

Violet

Online

200

50

50

50

Supermarket

65

20

21

21

Book Store

10

30

Total

275

100

80

80

Read the following statements:


Statement I: Online store accounted for 50% of the sales at BnC and the ratio of supermarket sales and
book store sales is 1:2.
Statement II: Initial Cost is allocated in the ratio of sales.
If you want to calculate the profit/loss from the different retail formats, then
1. Statement I alone is sufficient to calculate the profit/loss.
2. Statement II alone is sufficient to calculate the profit/loss.
3. Both statements I and II are required to calculate the profit/loss.
4. Either of the two statements is sufficient to calculate the profit/loss.
5. Neither Statement I nor Statement II is sufficient to calculate the profit/loss.

DIRECTIONS for questions 2 to 4: Based on your answer to the above question, answer for the questions that
follow.
2.

What is the profit/loss from "online" sales?


1. 0
4. + 450

3.

Which retail format is least profit making for BnC?


1. Online
4. All formats are loss making.

4.

2. -310
3. + 20
5. Cannot be determined from given information

2. Supermarket
5. All formats are profit making.

3. Book Store

Which retail format gives the highest profit for BnC?


1. Book Store
4. All are equally profitable.

2. Supermarket
3. Online
5. Cannot be determined from given information.

DIRECTIONS for questions 5 & 6: Read the following caselet and choose the best alternative for the questions
that follow.
A teacher wanted to administer a multiple choice (each question having six choices) based quiz of high difficulty
level to a class of sixty students. The quiz had sixty questions. The probability of selecting the correct answer for
a good student and a brilliant student was 0.2 and 0.25 respectively. The poor students had no learning
advantage. The teacher did not want students to cheat but does not have time and resources to monitor. All
students were seated serially in 10 rows and 6 columns.
5.

Is it possible for the teacher to detect cheating without monitoring? Choose the statement that best
describes your opinion:
1. It is not at all possible; teacher will have to introduce technology if there is no human support.
2. It is always possible; but teacher has to calculate the exact answer.
3. It is possible when many students sitting next to each other have the same incorrect answers for
multiple questions. However, there can be a small error in judgment.

` 29
4. It is possible when many students sitting next to each other have the same correct answers for multiple
questions. However, there can be a small error in judgment.
5. It is possible only for poor students but not for good and brilliant students. However, there can be a
small error in judgment.
6.

Students from four sections of a class accompanied by respective class teachers planned to go for a field
trip. There were nineteen people in all. However, on the scheduled day one of the four teachers and a few
students could not join the rest. Given below are some statements about the group of people who
ultimately left for the trip.
I: Section A had the largest contingent.
II: Section B had fewer students than Section A.
III: Section C's contingent was smaller than Section B.
IV: Section D had the smallest contingent.
V: The product of the number of student from each section is a multiple of 10.
VI: The number of students from Section C is more than 2.
VII. The product of the number of students from each section is a multiple of 24.
VIII. The largest contingent has more than 4 students.
IX: Each section contributed different number of students
The statements that taken together can give us the exact number of students from each section:
1. I, II, III, IV, VI

7.

2. I, VI, VIII, IX

3. I, II, III, IV, V, VI 4. I, II, III, IV, VII

5. I, IV, VI, VII, IX

There is a lot of interest in the first five ranks for Class XI students. One student guessed the rank order
as Ankita, Bhagyashree, Chanchal, Devroopa and Esha. Later upon announcement of the results, it was
found that not only did he get each student out off her true position, none of the students in his ranking
correctly followed her immediate predecessor. Another student guessed Devroopa, Ankita, Esha,
Chanchal and Bhagyashree. Even his guess was wrong. It was found that he had got two positions
correct, and two students in his ranking correctly followed their immediate predecessors. Which of the
following is true about the correct rank order?
1. Ankita got the third position
3. Chanchal got the second position
5. Esha got the fourth position

2. Bhagyashree got the fourth position


4. Devroopa stood first

DIRECTIONS for questions 8 & 11: Read the following caselet and choose the best alternative for the questions
that follow.
Krishnapuram's town council has exactly three members: Arjun, Karn, and Bhim. During one week, the council
members vote on exactly three bills: a recreation bill, a school bill, and a tax bill. Each council member votes
either for or against each bill. The following is known:
Each member of the council votes for at least one of the bills and against at least one of the bills.
Exactly two members of the council vote for the recreation bill.
Exactly one member of the council votes for the school bill.
Exactly one member of the council votes for the tax bill.
Arjun votes for the recreation bill and against the school bill.
Karn votes against the recreation bill.
Bhim votes against the tax bill.
8.

Which one of the following statements could be true?


1. Arjun and Karn vote the same way on the tax bill.
2. Karn and Bhim vote the same way on the recreation bill.
3. Karn and Bhim vote the same way on the school bill.

` 30
4. Arjun votes for one of the bills and Karn votes for two of the bills.
5. Arjun votes for two of the bills and Karn votes for two of the bills.
If the set of members of the council who vote against the school bill are the only ones who also vote
against the tax bill, then which one of the following statements must be true?

9.

1. Arjun votes for the tax bill.


3. Karn votes against the school bill.
5. Bhim votes against the school bill.
10.

2. Karn votes for the recreation bill.


4. Bhim votes against the recreation bill.

If Karn votes for the tax bill, then which one of the following statements could be true?
1. Arjun and Karn each vote for exactly one bill.
2. Karn and Bhim each vote for exactly one bill.
3. Arjun votes for exactly two bills.
4. Karn votes for the recreation bill.
5. Bhim votes against the recreation bill.

11.

If Karn votes for exactly two of the three bills, which one of the following statements must be true?
1. Arjun votes for the tax bill.
3. Karn votes for the school bill.
5. Bhim votes for the school bill.

2. Karn votes for the recreation bill.


4. Karn votes against the tax bill.

DIRECTIONS for questions 12 to 14: Answer these questions on the basis of the information given.
Dev Anand, CEO of a construction company, recently escaped a potentially fatal accident. Dev had failed to
notice a red light while driving his car and attending to his phone calls. His well-wishers advised him to get a
suitable replacement for the previous driver Ram Singh, who had resigned three months back.
Ram Singh was not just a driver, but also a trusted lieutenant for Dev Anand for the last five years. Ram used to
interact with other drivers and gathered critical information that helped Dev in successfully bidding for different
contracts. His inputs also helped Dev to identify some dishonest employees, and to retain crucial employees who
were considering attractive offers from his competitors. Some of the senior employees did not like the informal
influence of Ram and made it difficult for him to continue in the firm. Dev provided him an alternative job with one
of his relatives.
During the last three months Dev has considered different candidates for the post. The backgrounds of the
candidates are given in the table below.
Name

Age

Educational
Qualification

Experience

Expected
Salary (Rs.)

Sunder

32

Post graduate

Seven years
of driving
experience

18,000 per
month

Mani

23

Studied up to
Standard IX

One year

8,000 per
month

Chintan

44

Graduate

20 years

20,000 per
month

Remarks
Ex-employers are highly satisfied. Their
only concern is about his tendency to
switch jobs after every six months. Enjoys
the newness in every job but tends to lose
interest after six months. Not willing to
commit for any more than six months
Claims to have more than one year of
experience, but can't provide any certificate
to substantiate it. He has received a hike of
Rs. 2,000 last month on account of his
good performance as a driver.
Working as a driver for the last one year
after losing his previous job of a
stenographer. He has been forced to take

` 31

Bal
Singh

40

Literate

More than 20
years

15,000 per
month

Chethan

38

Standard XII

10 years

12,000 per
month

up the job of a driver.


Cousin of Ram Singh. Substituted Ram as
Dev's driver whenever Rani was on leave.
Currently working as a driver with Dev's inlaws. Strongly recommended by Ram. His
knowledge and contacts in the firm is as
good as Ram's.
Working as a temporary driver with Dev's
major competitor for the last three years.
The competitor has offered Chethan's
service to Dev on a temporary basis.
Chethan has also expressed his willingness
to work on a long term basis, provided he is
given an annual increment of Rs. 500,
which is reasonable as per the market
condition.

Dev is primarily looking for a stable and trustworthy driver, who can be a suitable replacement for Ram. His family
members do not want Dev to appoint a young driver, as most of them are inexperienced. Dev's driver is an
employee of the firm and hence the appointment has to be routed through the HR manager of the firm. The HR
manager prefers to maintain parity among all employees of the firm. He also needs to ensure that the selection of
a new driver does not lead to discontent among the senior employees of the firm.
12.

From his perspective, and taking into account the family's concerns, Mr. Dev would like to have
1. Chethan

13.

2. Chintan

3. Bal Singh

4. Mani

5. Sunder

In order to resolve the conflicting preferences, one of Dev's friends suggested Dev, his family members
and the HR manager to identify their most and the least preferred candidates without considering the
concerns of other stakeholders.
I. Dev's most and least preferred candidates: Bal Singh and Chetan respectively
II. Family members most and least preferred candidates: Bal Singh and Chintan respectively
III. HR managers most and least preferred candidates: Chethan and Bal Singh respectively
Which of the above three statements is/are in conformity with the information provided in the passage?
1. Option I
4. Options II and III

14.

2. Option II
5. Options I, II and III

3. Options I and II

Who among the following five candidates is most likely to be rejected by the GM (HR)?
1. Chethan

2. Chintan

3. Bal Singh

4. Mani

5. Sunder

DIRECTIONS for questions 15 to 17: Answer these questions on the basis of the information given.
Alex Company has its office at the third floor of a multi-storied building in Mumbai. There are 5 rooms to be
allotted to 5 managers (designated M1 to M5), each of whom will occupy one room. Each room has its own
advantages and disadvantages. Some have the sea view, while others are closer to either the lift or the dining
room, while some are more spacious. Each of the five managers was asked to rank the room preferences
amongst the rooms 301, 302, 303,304 and 305. Their preferences were recorded as follows:

` 32
Preference
1st
2nd
3rd
4th
5th

M1
302
303
304

M2
302
304
305
301

M3
303
301
304
305
302

M4
302
305
304
303

M5
301
302
305

Some managers did not indicate any preference for some rooms, as they did want to be there under any
condition. The company decided to allot rooms to managers in such a way that so that the managers get rooms
as per their best preference or close to that.
15.

How many managers would get their rooms as per their best preference?
1. 1

16.

3. 3

4. 4

5. 5

If manager X gets his/her 1st choice, then his/her preference ranking is 1 and so on. Management decided
to allot rooms so that the sum of the preference rankings of all the managers is minimized, what is the
total preference ranking for the rooms allotted to all the managers?
1. 5

17.

2. 2

2. 6

3. 7

4. 8

5. 9

Suppose that Manager M2 decides-not to join the new zonal office and Manager M6 takes his place.
Manager M6 has the following preference ranking in decreasing order: 301, 303, 302, 304 - in this case
what would be the sum of the preference rankings allotted to all the five managers?
1. 5

2. 6

3. 7

4. 8

5. 9

DIRECTIONS for questions 18 to 20: Answer based on the following information:


On a certain day six passengers from Chennai, Bengaluru, Kochi, Kolkata, Mumbai, and Hyderabad boarded the
New Delhi hound Rajdhani Express from TataNagar. The following facts are known about these six passengers:

The persons from Kochi and Chennai are less than 36 years of age.

Person Z, the youngest among all is a doctor.

The oldest person is from Kolkata and his/her profession is same as that of the person who got down at
Mughal Sarai.

The persons from Bengaluru, Chennai, Hyderabad and Mumbai got down at four different stations. The
eldest among these four got down at Koderma and the youngest at Kanpur. The person who got down at
New Delhi is older than the person who got down at Mughal Sarai.

The engineer from Bengaluru is older than the engineer from Chennai.

While arranging the teachers in increasing order of age it was observed that the middle person is as old
as the engineer from Chennai.

Person Y who got down at Mughal Sarai is less than 34 years old.

The teacher from Kochi is four years older than the 31 year old doctor who is not from Mumbai.

In the past, three of the travellers have served in the Indian Army.
18.

Which of the following options is true?


1. The person from Chennai is older than the person from Kochi.
2. The oldest teacher is from Mumbai.
3. The person from Mumbai is older than at least one of the engineers.
4. The person from Kochi got down at Mughal Sarai, and was an engineer.
5. The person who got down at New Delhi is older than Y, who in turn is older than the person from
Hyderabad.

` 33
19.

All six travellers are working in the same organization for at least one year. The organization recruits two
categories of employees - fresh graduates and those who have at least five years' of experience in the
Indian Army. In both cases a new recruit should be less than 30 years of age. Among the travellers from
Mini profession, those with military background are at least five years older than the travellers who joined
as fresh graduates. Identify the travellers who joined the organization as fresh graduate(s):
1. Only Y
2. The person Y and the traveller from Chennai.
3. The person Y and the travellers from Kochi and Hyderabad.
4. The travellers from Kochi and Hyderabad.
5. The teacher from Mumbai, the traveller from Kochi and the younger engineer.

20.

If W is neither the youngest nor the oldest among the travellers from her profession, which of the following
true about her?
1. She got down at Koderma.
3. She got down at Mughal Sarai.
5. None of the above

2. She is 36 years old.


4. She is from Kochi.

DIRECTIONS for questions 21 & 22: Answer bused on the following information:
Mrs. Sharma has a house which she wants to convert to a hostel and rent it out to students of a nearby women's
college. The house is a two storey building and each floor has eight rooms. When one looks from the outside,
three rooms are found facing North, three found facing East, three found facing West and three found facing
South. Expecting a certain number of students, Mrs. Sharma wanted to follow certain rules while giving the
sixteen rooms on rent:
All sixteen rooms must be occupied.
No room can be occupied by more than three students.
Six rooms facing north is called north wing. Similarly six rooms facing east, west and south are called as east
wing, west wing and south wing. Each corner room would be in more than one wing. Each of the wings must have
exactly 11 students. The first floor must have twice as many students as the ground floor.
However, Mrs. Sharma found that three fewer students have come to rent the rooms. Still. Mrs. Sharma could
manage to allocate the rooms according to the rules.
21.

How many students turned up for renting the rooms?


1. 24

22.

2. 27

3. 30

4. 33

5. None of the above

If Mrs. Sharma allocates the north-west corner room on the ground floor to 2 students, then the number of
students in the corresponding room on the first floor, and the number of students in the middle room in
the first floor of the east wing are:
1. 2 and 1 respectively
3. 3 and 2 respectively
5. Such an arrangement is not possible.

2. 3 and 1 respectively
4. Both should have 3 students

` 34
DIRECTIONS for questions 23 to 27: Each group of questions in this section is based on a set of conditions. In
answering some of the questions, it may be useful to draw a rough diagram.
Seven film buffs - Gangadhar, Indra, Lalatakshaya, Maheshwar, Rudra, Vyomkesha, and Yogi - attend a showing
of classic films. Three films are shown: one each directed by Guru Dutt, Satyajit Ray, and Ritwik Ghatak. Each of
the film buffs sees exactly one of the three films. The films are shown only once, one film at a time. The following
restrictions apply:
Exactly twice as many of the film buffs see the Satyajit Ray film as see the Guru Dutt film.
Gangadhar and Rudra do not see the same film as each other.
Indra and Maheshwar do not see the same film as each other.
Vyomkesha and Yogi see the same film as each other.
Lalatakshaya sees the Satyajit Ray film.
Gangadhar sees either the Guru Dutt film or the Ritwik Ghatak film.
23.

Which one of the following could be an accurate matching of film buffs to films?
1. Gangadhar: the Satvajit Rav film; Indra: the Ritwik Ghatak film; Maheshwar: the Satyajit Ray film
2. Gangadhar: the Ritwik Ghatak film; Indra: the Guru Dutt film; Vyomkesha: the Guru Dutt film
3. Indra: the Satyajit Ray film; Rudra: the Ritwik Ghatak film; Vyomkesha: the Guru Dutt film
4. Maheshwar: the Ritwik Ghatak film: Rudra: the Ritwik Ghatak film; Vyomkesha: the Ritwik Ghatak film
5. Maheshwar: the Satyajit Ray film; Rudra: the Satyajit Ray film: Yogi: the Satyajit Ray film

24.

Each of the following must be false EXCEPT:


1. Rudra is the only film buff to see the Guru Dutt film.
2. Rudra is the only film buff to see the Satyajit Ray film.
3. Yogi is the only film buff to see the Ritwik Ghatak film.
4. Exactly two film buffs see the Ritwik Ghatak film.
5. Exactly three film buffs see the Satyajit Ray film.

25.

If exactly one film buff sees the Ritwik Ghatak film, then which one of the following must be true?
1. Vyomkesha sees the Satyajit Ray film.
2. Gangadhar sees the Guru Dutt film.
3. Maheshwar see the Guru Dutt film.
4. Indra sees the Guru Dutt film.
5. Rudra sees the Satyajit Ray film.

26.

If Vyomkesha and Gadhadhar see the same film, then which one of the following could be true?
1. Gangadhar sees the Guru Dutt film.
2. Indra sees the Satyajit Ray film.
3. Rudra sees the Ritwik Ghatak film.
4. Vyomkesha sees the Satyajit Ray film.
5. Yogi sees the Guru Dutt film.

27.

Each of the following could be a complete and accurate list of the film buffs who see the Guru Dutt film
EXCEPT:
1. Gangadhar, Indra
4. Maheshwar, Rudra

2. Gangadhar, Maheshwar
5. Vyomkesha, Yogi

3. Indra, Rudra

` 35
28.

Five persons - A, B, C, D and E - are either guards or thieves. The guards always tell the truth, whereas
thieves always lie. A claims that B is a guard. B claims that C is a thief. C claims that D is a thief. E claims
that A is a guard. D claims that B and E are different kinds. The number of thieves is
1. 1

2. 2

3. 3

4. 4

5. 5

DIRECTIONS for questions 29 to 31: Each group of questions in this section is based on a set of conditions. In
answering some of the questions, it may be useful to draw a rough diagram.
A company launches eight products - Q, R, S, T, V, W, Y, and Z - in one of the four metros of India. The products
were launched one after the other over a period of six months in 2006. The order in which the products were
launched is consistent with the following conditions:
V is launched before both Y and Q
Q gets launched after Z
T gets launched before V but after R
S gets launched after V
R gets launched before W.
29.

Which one of the following could be true?


1. Y is the second product to be launched.
3. Q is the fourth product to be launched.
5. V is the sixth product to be launched.

30.

If R is the second product to be launched, which one of the following MUST be true?
1. S gets launched some time before T.
3. W gets launched some time before V.
5. Z gets launched some time before W.

31.

2. R is the third product to be launched.


4. S is the fifth product to be launched.

2. T gets launched some time before W.


4. Y gets launched some time before Q.

If V gets launched before Z does, then which one of the following COULD be true?
1. R is the second product to be launched.
3. Q is the fourth product to be launched.
5. Z is the sixth product to be launched.

2. T is the fourth product to be launched.


4. V is the fifth product to be launched.

DIRECTIONS for questions 32 to 34: Each group of questions in this section is based on a set of conditions. In
answering some of the questions, it may be useful to draw a rough diagram.
A famous retail electronics showroom chain has six new mobile phone models - T, V, W, X, Y, and Z each
equipped with at least one of the following three options: digital camera, music player, and office document
viewer. No mobile has any other option. The following conditions apply:
V features both a digital camera and an office document viewer.
W has digital camera and music player
W and Y have no options in common.
X has more options as compared to W
V and Z have exactly one option in common
T has fewer options as compared to Z.
32.

For exactly how many of the six mobile phones is it possible to determine exactly which option each one
has?
1. Two

2. Three

3. Four

4. Five

5. Six

` 36
33.

Which one of the following must be false?


1. Exactly five mobile phones feature a music player.
2. Exactly five mobile phones feature a document viewer.
3. Exactly four mobile phones feature a music player.
4. Exactly four mobile phones feature a digital camera.
5. Exactly four mobile phones feature a document viewer.

34.

If exactly four of the six mobile phones have music player, and exactly four of the six mobile phones have
digital camera, then each of the following must be true EXCEPT:
1. T and V have no options in common.
2. T and Y have no options in common.
3. T and Z have exactly one option in common.
4. W and Z have exactly one option in common.
5. Y and Z have no option in common.

` 37

Answer keys & Explanations Quant & DI


Actual Previous Year XAT
1.
2.
3.
4.
5.
6.
7.
8.
9.
10.
11.
12.
13.
14.
15.
16.
17.
18.
19.
20.
21.
22.
23.
24.
25.
26.
27.
28.
29.
30.
31.
32.
33.
34.
35.
36.
37.
38.
39.
40.
41.
42.
43.
44.
45.
46.
47.
48.
49.
50.

1
3
5
4
1
5
4
4
1
2
5
3
5
3
2
3
5
2
4
1
1
3
3
5
3
1
4
4
1
2
2
2
5
3
1
3
1
5
4
2
4
5
3
3
3
3
1
1
4
3

51.
52.
53.
54.
55.
56.
57.
58.
59.
60.
61.
62.
63.
64.
65.
66.
67.
68.
69.
70.
71.
72.
73.
74.
75.
76.
77.
78.
79.
80.
81.
82.
83.
84.
85.
86.
87.
88.
89.
90.
91.
92.
93.
94.
95.
96.
97.
98.
99.
100.

2
4
5
4
2
3
3
4
1
5
4
5
4
4
3
1
4
1
1
4
1
5
3
3
2
3
1
1
4
3
5
4
1
1
2
1
5
1
4
4
1
5
3
3
2
4
3
1
2
4

101.
102.
103.
104.
105.
106.
107.
108.
109.
110.
111.
112.
113.
114.
115.
116.
117.
118.
119.
120.
121.
122.
123.
124.
125.
126.
127.
128.
129.
130.
131.
132.

5
4
4
5
1
4
2
1
2
2
2
4
1
4
2
2
3
1
4
1
1
4
3
2
4
3
2
4
1
3
3
2

` 38
1.

3y
3y
y
78 y = 24.
we get
4
2
2.

3.

For first 2 km, the fare is 2 10 = Rs 20, for next 8 kms, the
fare is 8 5 = Rs 40, for next 4kms, the fare is 4 8 = Rs 32.
So total fare = 20 + 40 + 32 = Rs 92
So amount left is Rs 10 and the remaining distance to be
covered by Bullock = 10/2 = 5km
Thus total distance = 14 + 5 = 19km

We have
(a 2 a 1)(b 2 b 1)(c 2 c 1)(d 2 d 1)(e 2 e 1)
abcde
1
1
1

a 1 b 1 c 1
a
b
c

1
1

d 1 e 1 .............................(i)
d
e

Now for positive numbers, a +


a+

1
2
a

1
+13
a

Similarly b +

1
1

+1 3, c 1 3 ..
c
b

(i)
(a 2 a 1)(b 2 b 1)(c 2 c 1)(d 2 d 1)(e 2 e 1)
3
abcde

3 3 3 3 = 243.
4.

5.

I 100 13.5 100

20
PT
7.5 9

Let x, y and z be the no of pages read by Albela, Bob and


Chulbul respectively. Also time taken by each of them is
equal i.e. 2x = 3y = 4z. Since total pages are 78, which
means
x + y + z = 78 . Putting the values of x and z in terms of y,

400

I 100 13.5 100

30
PT
7.5 6
7.

8.

9.

As the sum of all six digits is 90 and the median is 18.


Therefore, the first two numbers will be 1 and the next two
will be 17 and 19 ( as median is 18 ). The next number
should be 20 and sum up to here is 58. Hence the largest
number is 90 58 = 32.

From the formula, total approx loss = (20/10)2 = 4%.


Now CP 0.96 = 60000 CP = Rs 62500. Hence the
loss is Rs 2500.
Let the failed candidates be F and the total candidates
be 100. Then,
(100 F).x% + F.y% = z%(100). Solving the equation,
we get 100x Fx + Fy = 100z;
Hence F = 100(z x)/(y x)%.

10. Arun can do a maximum of 3 tasks. So as per priority and


no. of days required, he should first do T1, and then T2 as it
has higher priority and last must be T5, not T4 as no of days
required in T4 are more, thus option 2
11. Arun can do a maximum of 3 tasks. So as per priority and no
of days required, he should first do T1, next he cannot take
T2 as it has highest number of background research days so
he will take next two tasks T4 and T5 as per their priority
number, thus option 5
12. Let the radius of the innermost circle be 1. Then radius

of successive circles will be 2 and 3. First area = (2)2


- = 3. Second area = (3)2 - (2)2 = 5. Reqd ratio
= 3/5.

13. Probability (a) = 1 king & 1 not king OR Both kings =


2

Since both ways Auto takes 30 minutes so one way Auto


takes 15 minutes. Now Walking and Auto takes 90 minutes,
which implies 90 15 = 75 minutes is for Walking. So, both
ways Walking would take 75 2 = 150 minutes.
Let point A be the first edge where the fish is released and B
be the point where they divert after travelling for 300 mtrs.
and C be the point where they reach another edge after
travelling for 400 mtrs. Now angle between AB & BC to be
90. Then AC must be 500 mtrs. and hence the diameter of
the pool which gives the radius to be = 250 mtrs. Now, area
will be r2 = (250)2 = 62500
B

Similarly, for older son, rate =

C1 4 C1 2 C 2
6

500

A
6.

Here P = 15/2 = 7.5 lakhs and Simple interest = 21 7.5 =


13.5 lakhs
Therefore, for younger son, rate =

9 3
.
15 5
4

Probability (b) = Not drawing a king =

C2
C2

6 2
.
15 5

Now ratio of a:b = 3/5 : 2/5 = 3/2, which is greater than 1.25.
14. In the figure PQRS, PQ = 6, PA = 4 so QA = 213(By
Pythagoras) and AB = 22. Now for area of ABQ we need
the height which will be
(213)2 = x2 + (2)2. Solving x = 50. Now area = Base
Height;
22 50 = 10.

P
300

C2

4
A
2
S

15. Since in every interval of 100 numbers we have 19 numbers


which have a particular digit used so from 100 to 800 we

` 39
have 6 (except from 200 to 299 which has all 100 numbers
having 2 in them) groups of 19 each. So, the total numbers
will be 19 6 = 114+100 =214.
16. Since we need to maximize the relation so we try to equalize
the variables p, q & r and also their sum is . This happens at
3+3+4=10. Now putting values we get 9+12+12+36=69
which lies in the range 60 to 70.
17. Let the number be xy.

x + y + xy = 10x+ y
x+ xy = 10x
1 + y = 10 y = 9
Hence numbers are 19, 29, 39, 49, 59, 69, 79, 89, 99
9
1
= 0.09.

99 11
Totaling the percentage of players who do not play each of
the games we get 30+25+20+15=90. Now the minimum
percentage of students who must be playing all the four
games will be 100-90=10.
Pq = qp and q = 9p, putting the value of q in first equation, we
get p9p=(9p)p p9p = 9p.pp p8p = 9p Taking 1/8p root
on both sides, we get p = 91/8
Since the options are whole numbers lets assume that Hari
got off the car after 3 hrs. (as 1, 2 and 4 hrs. is not possible).
Now that means Ram & Hari must have travelled for 25 3
= 75 kms., so 25 kms. still remain to be covered which Hari
will do at 5 km/h in 25/5=5 hrs. Now in those 5 hrs. Ram can
reach back for Shyam and take him to the destination in
those 5 hrs.
Let x be the width of the roads. As length and breadth are
40m and 30m respectively
Now as per question, 40x + 30x x2 = 600 70x x2 =
600 x2 70x + 600 = 0
x = 60 or 10. As 60 is not possible, therefore width of the
road is 10m
The road connecting all the cities should be the diagonals
from A to C and from B to D. Since the distances between
A,B,C&D are 10 each so the diagonals must be 102
each(By Pythagoras). So, the total length should be 2(102)
= 202 = 20 1.414 = 28.3 approx.
Statement I is wrong as 19 is also one of the prime factor,
Statement II is also wrong.
In Statement III, let the average speed for the whole journey
is 2x. Let the speed in the second part is a km/hr. Now

In Statement III: Let the imports of goods and services in


2004 = 100
imports of goods and services in 2010 = 100 1.1799
1.09438 1.163 1.1084 1.08321 1.1649 = 210
Let the exports of goods and services in 2004 = 100
Exports of goods and services in 2010 = 100 1.1888
1.1383 1.1713 1.1063 1.00813 1.2186 = 215.4
Hence, exports increased at faster rate than imports during
the period 2005 to 2010.
26.
Population in 2006 =

= 1.12 109
= 112 107
= 1120 million.

Required fraction is

22.

23.

2ax
2 x a a x x 0 which is not possible.
ax
Hence statement III is true.
24.

Current balance in 2010 =

73555.34

3.268
100
=-

2403.78billion Rs.
Current balance in 2005 =

35662.2

1.272
100
= - 453.62

2403.78
5.30
Required Ratio =
453.62
25. Nothing can be said about the export and import, Hence
nothing can be said about the first two statements.

Lost

Points

GF

GA

Goal Diff
=GF GA

Rank

27- The following table can be formed for the AWAY matches:
30.
Draw

21.

8.9
99.6million 100 million.
100

Won

20.

1120

19.

Unemployed in 2006 =

Team

18.

41159.73 109
36553.93

CH

SW

WB

MC

MU

WH

-3

10

EV

AS

NC

-2

FU

-4

11

ST

WG

SU

Information cannot be determined

Information cannot be determined

27. According to the above table, it can be seen that based upon
AWAY matches ranking, team SW will be the second
ranking team.
28. At least these teams have 0 or 1 points WB, MC, WH, NC
& FU. These are 5 in number.
29. The pos table for the answer options can be formed as
follows:
Team
Home Rank
Away Rank
pos
AS

12

11

WB

-7

WH

10

-7

MC

-3

SW

` 40
Thus, maximum value of pos is for the team AS i.e. 11.
30. These are different values possible for Goal differences: 2, 5,
0, - 3, - 2 and 4. These are 6 different values.
31.
10
15
0.2. Potency 2:
2 0.6.
Potency 1:

50
50
30
4 2.4. So total dose = 0.2 + 0.6 + 2.4 =
Potency 3:
50

(v) a+ b = 4
(vi) Jose has fathered a twin so two ages are same.
Using above we can find c = 9 and a =b =2 years. So ages
are 2, 2, 9 years.
39.

2
0

1
10 n
10 10
=> n = 2.9
2
10
(approximately). Hence option 4.

II 12

15

3.2. Hence option 2.


32.

Solution Re maining

TotalSolut ion

Final vol. = Initial

40. Since the difference in angles is 90, these angles are

135 and 45 respectively. Taking one triangle which is


half the rhombus, it is an isosceles triangle with angle
45 and 2 sides of 2 each. Area = bc sin A.

3
1

Hence area of rhombus =


1
2 2 2 sin 45 0 2 2cm 2 .
2

III 8
So required total = 9 + 3+ 1 =13. Hence option 2.
33. No information is given about 2010. Hence data is
insufficient to solve the question.
34. Revenue of Yahoo in 4th year = $250m and in 6th year is
$1150 m
Avg. percentage increase in revenue of Yahoo
=

900 100

180
250
2

Revenue of Facebook in 4th year = $350m and in 6th year is


$2000 m
Avg. percentage increase in revenue of Facebook

1650 100

235
350
2

So diff.=235 180 = 55%


35. Percentage increase in the revenue of goggle from 5th to 6th
year =

3250 1500
1750
100
%.
1500
15

So, the Facebook revenue in sixth revenue should be

3250
750 1625.
1500

36. VP (HR) visits every 3rd day, VP (Operations) visits every 4th
day and VP (Sales) visits every 6th day. They will meet
together after LCM of 3, 4, and 6 = 12 days. So after January
3, they should meet on January 15 and next on January 27.
But CEO is on leave till Jan 27. So they will meet after 12
more days i.e. February 8, 2012. Hence option 3.
37. Mean of five Rs 5 lakh. So their total is 25 lakh. Median
salary is Rs 5 lakh. So two salaries are less than 5 lakh and
two salaries are more than 5 lakh. Mode is 8 lakh. So there
have to be two salaries of 8 lakh. Hence three salaries are 5
lakh, 8 lakh and 8 lakh. Now other two salaries total = 25
(5+ 8 + 8) = 4 lakh. Only possibilities are (1, 3) and (2, 2)
but (2, 2) is not possible as there is only one mode. So five
salaries are 1, 3, 5, 8, and 8. So answer is 1 + 8 = 9. Hence
option 1.
38. (i) Let ages of three sons be a, b and c.
(ii) a + b+ c = 13

41.

Volume of given cone=

22 3 22 3
cm .
cm of volume is used
48
48

to write 330 words. Hence 600cm3of volume will write

48
330 600 432000 words
22
42. Since water lost by each sister is same. So the %age lost will
be maximum for the sister who has minimum capacity. So
answer is Bina.
43. Let the first term is a and common difference is d:

S1 = a and S4 = a + a + d + a + 2d +a + 3d = 4a + 6d.
Given that

S1
1
a
1

S4 10
4a 6d 10

10a + 4a+ 6d
a = d.
a
a
1

a 3d 4a 4
44. At the end of first year the interest is Rs 300. So total
payable amount is Rs 6300 out of which he has paid Rs
1200. So balance amount is Rs 5100. In the second years the
interest is Rs 255. So payable amount is Rs 5355. He has
again paid Rs 1200. So the remaining amount is Rs 4155.
45. As the total volume remains the same so radius of smaller
sphere will be 1cm. As surface area is proportional the
square of radius. So total surface area of 1000 smaller
spheres will become 1000 12 =1000 and surface area of one
big sphere will become 102 = 100. Hence total surface area is
increased by 900% i.e. 10 times.
46. Let the pots are x, so the total amount is x2.
Now if one brother is having potato chips and banana chips
and the other has only potato chips packets and the number
of packets with each brother is same, so the no. of packets of
potato chips is an odd no.
Let the no. of packets of potato chips is y, so we have 10y +
B = x2
Where B is the price of the packet of banana chips.

the required ratio =

` 41
Only one case is possible. They collected total money Rs.36.
In this case they will buy 3 packets of potato chips and one
packet of banana chip. So one will get two potato chips
packet worth Rs.20 and other will get one packet of banana
chips and one packet of potato chips i.e. total worth Rs.16.So
one should give Rs.4 to the other to make the division
financially equitable.
47. We have K + G + S = 100 (1)
10K + 3G + S = 100
20k + 6G + S = 200 (2)
Subtracting (1) from (2), we get 19K+ 5G = 100 .. (3)
G = 1 and K = 19 is the only solution possible for equation
(3)
No. of Gulabjamuns bought by Nikhil is 1.
48. Total petrol used during journey = 10+ 20+ 15 + 10 5 = 50
liter. So the mileage of the car =

400
8kmpl.
50
49. Since petrol in Rampur is Rs 45/ liter, she will buy it from
Rampur just to reach 650 km mark i.e. 18.75 liter. But she
already has 5 liter so she will buy only 13.75 liter. So cost is
13.75 45 = 618.75. Next she will buy 6.25 liter to reach
600 km mark @ Rs 40/ litre. So cost here will be 40 6.25 =
Rs 250. Here she will buy 25 litre petrol @ Rs 35/ litre
which will cost
35 25 = Rs 875. So the total cost = 618.75 + 250 + 875 =
Rs 1743.75. Hence option 4.
50. Here the paths are AD, AE and AF. Since E is the midpoint
of hypotenuse,
It is equidistant from all the vertices. So AE = 40. Now AE
divides DF in two equal parts so it is the median to DF. By
Apollonius theorem, AF2 + AD2 = 2(AE2 + EF2)
AF2 + AD2 = 2(1600 + 400) = 4000
So the sum of squares of paths = AF2 + AD2 + FD2 = 4000 +
1600 = 5600m

C
20

replaced =

1
27 9 litre.
3

52.

A
7 km

B
4 km

When the two buses meet for the first time, the bus A has
covered 7 km and the two buses has covered one complete
distance. By the time they meet again at D, the two has
covered total distance thrice in which the contribution of bus
A is 7 3 =21 km. But in actual the bus from A has covered
one full distance plus 4 km. So the distance between A and B
= 21 4= 17 km. Now each bus makes 5 rounds per day i.e.
34 5 = 170 km. So cost of running the buses = 170 20 2
= Rs 6800.
53.

50

50

50

50

Since FCD = 450. So CD = FD = 25


50.

2 = BE. Also DE =

100 25 2 25 2
Now AB2 =
2

which on

5 8.

solving gives AB = 50
=> a = 50, b = 5, c =8.
So a +b +c = 50 + 5 + 8 = 63. Hence option 5.
54. 10 roti+4 tadka+1 tea=80(1)
7roti+3 tadka+1 tea=60(2)
Multiplying 1st equation by 2 and 2nd equation by 3 and
subtracting equation 1 from 2 we get 1roti +1 tadkas +1
tea=20 Rs. So 5roti +5 tadkas +5 tea=100Rs.
55.

F
20

3
E
20

40

D
E
B

51. By allegation method-

12%

39%
21%

18

1,

1
rd of 27lt=9lt
3

The part replaced is 1/3rd of total mixture. Hence the part

2
20

2
2

This question can be done by reverse approach. As perimeter


of remaining land (i.e. land left after selling two square
plots) to original is 3:8. So we took the dimensions of
smallest plot as 2 and 1 i.e. AD as 2 and AB as 1. As
remaining plots are square. So remaining dimensions
become as shown in figure. Also the Ratio of perimeter of
Remaining plot to original plot becomes 6:16 i.e. 3:8. So
Now we can solve the SP per sq ft. of remaining plot. Total
Area of plot = 35=15 sq. feet
So cost @1000 per sq. feet = 151000=15000Rs.
Profit
= 10%
So total SP of entire plot = 150001.1
= 16500
16500 = 91200 + 4 1150 + 2 y

` 42
Where y is SP/sq. ft of remaining plot. On solving we get y =
550Rs.
56.

The no. of runs scored by 4s =

3.92
306 = 12
100

19039.55
100 5.64%.
337425
69.

No. of fours = 3.

317910
Rs 308650 .48
1.03

43 .14
The no. of runs scored by 6s =
306 = 132
100

Tax paid on income upto Rs. 5,00,000/- = 10% of 1,40,000 +


20% of 2,00,000 = 14000 + 40000 = 54000
Therefore, remaining tax must be on salary above Rs. 5 lacs.
Remaining Tax = Rs. 308650.48 Rs. 54000 = Rs.
254650.48/This remaining tax has been paid @ 30%.
Thus, salary above Rs.5 lacs =

No. of sixes = 22
57. Total score = 306 + 20 4 = 322
Runs scored by 4s = 12 + 20 = 32
Which is 9.94% of total score
Hence the central angle =

360
9.94 = 35.78
100

58. Check by option.acc to given condition option D will give


shyam a profit of 367000.profit of shyam -till 2L =120,000.
From 2L to L=40,000+96000=136,000.From 5L to
5.5L=52,500+58,500=111,000
Total profit of shyam=367000.
59. The values for CSR for the years 2005, 2006, 2007, 2008
and 2009 are 1.15 lakh, 2.00 lakh, 2.01 lakh, 2.47 lakh and
3.16 lakh respectively. Since the question asks to calculate
the maximum increase w.r.t. previous year, thus the answer
will be year 2006 as there is an increase of 0.85 lakh.
60. Since we need to calculate CSR/Assets and CSR = Assets
Sales % CSR spending %. Therefore, required ratio =
Sales % CSR spending %. The maximum value is for the
year 2008 and it is 1.65
61. The maximum value of spending on CSR activities is for the
year 2009 and it is 3.16 lakh.
62. The values for CSR for the years 2005, 2006, 2007, 2008
and 2009 are 1.15 lakh, 2.00 lakh, 2.01 lakh, 2.47 lakh and
3.16 lakh respectively. It can be checked that there is no
decline for the spending on CSR as compared to previous
year.
63. The percentages of Ex-defense servicemen for the given
levels are 11.53, 12.3, 14.28, 19.23 and 18.18 respectively.
Thus, the highest value is for Level-4.
64. If the present staff strength is compared with the requirement
at each level, it can be checked that vacancies exist for
Levels-1, 3 and 4 only. It can be visualized easily that the
maximum reduction (in % terms) will be for Level-4.
65. It can be checked that the lowest representation (in % terms)
of Ex-policemen is for level-3 and it is 4.28%.
66. First of all, the houses in which stealing is not done, can be
arranged in 1 way as O O O O O O O
Now, the 3 houses (in which stealing can be done) can be
arranged at any of the 8 places marked with X:
XOXOXOXOXOXOXOX
We can choose these 3 houses out of 8 places as = 8C3 = 56
ways
67. Every number of the type a6 = 111111 is divisible by 3, 7,
11, 13 & 37.
a741 is divisible by 3 and 37; a534 is divisible by 3, 37, 11,
13 & 37; a123 is divisible by 3 and 37; and a77 is divisible
by a7 and a11. Therefore, none of them is prime.
68. Income Tax payable = 10% of 1,10,000 + 20% of 37425
= 11000 + 7485 = 18,485
Total tax payable (Incl. Cess) = 1.03 X 18485 = 19039.55
Thus, percentage income payable as tax =

Total TDS = Rs. 3,17,910/Thus, Income tax before Cess =

254650.48

70.

71.

100
Rs 848835.
30

Therefore, total salary = Rs.8,48,835 + Rs. 5,00,000 = Rs.


13,48,835/We have Z = 21 sinX + 72 cos X = 3 [7 sinX + 24 cos X] Let
7 = r.cos, 24 = r.sin. Thus, r = 25.
And Z = 3 [25sinXcos + 25cos Xsin] = 75 sin(X + )
Since sin(X + ) can assume the maximum value as 1,
therefore the maximum possible value for Z will be 75.
P (Crime happening in Northern region)
= P(Major crime) + P(Minor crime) P(Both major and
minor crimes) = 0.418 + 0.612 0.418 0.612 = 0.7742
Similarly, P (Crime happening in Southern region)
= P(Major crime) + P(Minor crime) P(Both major and
minor crimes) = 0.355 + 0.520 0.355 0.520 = 0.6904
Therefore, P(Crime in Northern or Southern region)
= P(Northern region) + P(Southern region) P(Both
Northern and Southern region) = 0.7742 + 0.6904
0.7742X0.6904 = 0.9301
Thus, P(No crime in any neighborhood) = 1- 0.9301 =
0.0699

72.

O
B

B1

900

A
C

6
A1

300

24

30

C1

300 2 x 2

300

A1

` 43
Let OP be the tower of height h
AB be first position of man with shadow AC = 24m
AB be 2nd position of man with shadow AC = 30m
AB = AB = 6m and AA = 300m
Let OA = x

77.

F
D
48

BF
20

h x 24
OP OC


6
24
POC BAC AB AC

8
16

29
4

12

4h - 24 = x

OP
OC'
h OA'30


A
'
B
'
A
'
C
'
6
30
POC~ BAC

Also

73.

2
2
5h = x 300 30
Now (5h 30)2 = (4h 2h)2 + 3002
h = 106m.
Let the distance is x. In POC/ and B/A/C/, we have
106 x 30

6
30
x 30
5
106 =

530 = x +30
x = 500m.

74.

80.

The equation for the triangle after shifting the ladder is:
(7 + x)2 + (24 x/2)2 = 252 x = 8
Total time available to catch the bus is 32 minutes out of
which 20 minutes are required to walk to the bus stop. Thus,
12 minutes are available for checking the machines. Since it
takes 6 minutes to identify whether a machine is defective or
not, therefore, only 2 machines can be checked in the
available time.
Now, 2 machines out of 4 can be chosen in 4C2 i.e., 6 ways.
And only 1 way is favorable to choose both the defective
machines.
Required Probability = 1/6.

10

4 2 21
12

3
4

81. When n = 0,

n
n
0; when n 10,
1.
20 n
20 n
When n = 16,

n
n
4; when n 18,
9.
20 n
20 n
82.

R6
F

R4
G

R2
R4

R3

R3

R1

R1

8
600

1
100 33 km.
The least distance = 3

6 6 2
=3
Clearly, p = 3, q = 4.

25
7

Since 59 students do not study any subject, so the total


students = 181. Let total students who study BF only = x. So
x + 20 + 8+16+ 12 +4 +92 = 181. => x =29.
78. From statement I = 999
PQ.RQ = 9 111
9 3 37 = 27 37
P+Q+R+X
Sum of 2 + 7 + 3 + 7 = 19
Answer can he solved only through statement I
So, answer is (A) option.
79. Let p = 3 and q = 4and the distance covered in 4 days

24 - x/2

76.

S 124
A

25

x/2

75.

92

R3

R2

R2

We know that, in a circle: TQ x TP = TR x TS 8 x 18 = k


x (k+7) k = 9
Now, Area of PTS = ab(sin600)/2 = 18.16.(3/2)/2 = 723

R5
R1, R2 need to be blocked only.
83. A0 = 1994
A1 = f(A0) = f(1994) = 1995
A2 = 1996, .............. A6 = 2000
A7 = 200, A8 = 20, A9 = 2
Hence n = 9.

` 44
84. For any cubic expression ax3+bx2+cx+d=0,let , , be the
roots then

88.

b
(sum of the roots taken one at a time)
a
c
+ += (sum of the roots taken two at a time)
a
d
(product of roots )
=a

Slope of

++=-

1
, b3
200

For x = 0
y=3
86.

1
1
1
a 2 a b 2 AP b PD
2
2
2
2a
2a
, PD
b2
b
2a
2a
AP PD a

b2
b
2b 2( b 2 )
1
b( b 2 )
AP

Slope of
Equation of PB is (y 6) = -2x
Y = -2x + 6
Equation of RC is (y 2) =

2
x
9

2x
y
2
9
Point of intersection is

2x 6

2x
2
9

8
1
4 2 x 1 2 x
9
9

3
9
,y
2
4
3
2
Slope of AX = 2
9
3
4

89. Let the total number of days = x. Since 9 mornings were


clear, so there was rain in x 9 mornings. Similarly there
were x 12 evenings when it rained. We have x 9 + x 12
= 13 2x = 34
x = 17. Hence the rainless days = 17 13 = 4
90.
10
Average of a, b, c, d, e is
2

C3

10

C3

20
1 5
1
120
6 6

92. Statement I: Sum of two sides in a triangle should be greater


than third side i.e. a + 2b < a + a + b
a + 2b < 2a + b

6 36 16
3 5
2

a
1
b< a b

If b = 3 + 5 then AQ = 1 + 5
If b = 3 - 5 then AQ = 1 Which is a negative number

2
9

So the least possible value of X will occur when all the terms
have equal values.
So take a = b = c = d = e = 2 and the minimum value of x =
4.
91. Prob. of at least one junior professor = 1 (no junior) =

b2 2b = 4b 4
b2 6b + 4 = 0

6
2
3

RC

applying above in the equation x3 + px + q we get


++ = 0
+ + = p
we know that
(++)2 = ( 2+2+2)+ 2(+ +)
0 =( 2+2+2)+2p or
2+2+2=-2p .This is possible only when p is negative. As
the question is asking about the possible value and out of the
choices given only first option has a negative value. Hence
it is the answer.
85. Let y = ax + b
Where y is no. of accidents, x is overtime hours while a & b
are constant
Then 8 = 1000 a + b
5 = 400 a + b

PB

Hence it is not sufficient. Statement II tells that a and b are


positive, so it is also not sufficient. Even if we combine the
two statements, the question cannot be solved.

So AQ = 1 + 5
87. Let x, y, z number of portfolios 1, 2, 3 are purchased
2x + 4y + 2z = 12
x + 2y + 2z = 6
3y + 3z = 6
y+z=2
x = 2, y = 2, z = 0
(5) is the right option.

93.

AD2 = (AB)2 + BC
2

CE 2 ( BC) 2 AB
2

5
AD 2 CE 2 AB 2 BC 2
4

` 45
5
AC 2 AC
4

49 25

74 4
5

94. The two statements are not individually sufficient.


Now if we combine the two statements we get, E > B > D >
A > C.
Hence C is the smallest integer.
95. x2 y2 = 0 x2 = y2
(x a)2 + y2 = 1
x2 2ax + a2 + x2 = 1
2x2 2ax + (a2 1) = 0
For real solution
(2a)2 4(2) (a2 1) > 0
4a2 8a2 + 8 > 0
8 > 4a2 a2 < 2
As a is non-negative
a = 1 satisfies the condition.
96. For n = 2
A3.A1 = A22 1
A3 = (2)2 1 = 3, 5
A4.A2 = A32 1
(12) . (2) = A32 1
A3 = 5 only
A5.A3 = A42 + 1
A5(5) = (12)2 + 1
A5 = 29
A6.A4 = A52 1
A6.(12) = (29)2 1

A6

840
70
12

A7.A5 = A62 + 1
A7(29) = (70)2 + 1

A 7 (29)

4901
169
29

97. He will be eligible if he is upgraded to gold tier which needs


5 flights or journey of 12,500 km. in last 6 month or 12
month period.
Mr. Kakkar will be taking his fifth flight in March
Distance = 711 + 711 + 1407 + 1959 + 1081 < 12,500
98. Mr. Kakkar will be shifting to platinum when he completes
10 flights after having the gold status. After upgrade to gold
for fifth flight in March, he takes the 10th flight in
November. So total no. of months from March to November
= 7.
99. First-tier upgrade takes place in February while last upgrade
is in November.
Total no. of full months = 8
100. Following 12 month upgrade system, he would have reached
Gold tier in September.
101. Contribution by Airline tickets (25 20) is the highest.
102. Total revenue pre increase = 37 + 32 + 28 + 31 + 44) 5 (17
+ 24 + 22 + 21 + 25) 10 + 115 20 + (20 + 23 + 22 + 9 +
6) 15 = $5450 m
Net revenue from air ticket after price & volume hike = (25
+ 21 +16 + 16 + 37) 1.2 25 = 115 1.2 25 = $3450 m
Earlier, revenue from air tickets = 115 20 = $2300 m

% increase =

1150
100 21%.
5450

103. Let net sales be 100 in 2004


Then net sales for 2005 = 119.9
Increase in PAT = 119.9 6.9/100 6 = 8.27 6 = 2.27

2.27
100 37.8 %
% increase 6
104. In 2000, let total sale = 100.
Salaries and wages in 2000 = 5.9
In 2001, the sale = 119.3 and salaries and wages are 6.8, so
0.9
the %age change = 5.9 100 = 15.25%
In 2002, the sale = 122.4 and salaries and wages are 5.6% of
0.05
122.4 = 6.85, so the %age change = 6.8 100 = 0.73%
In 2003, the sale = 141.6 and salaries and wages are 5.3% of
141.6 = 7.5
0.65
The %age change = 6.85 100 = 9.48%
In 2004, the sale = 163.1 and salaries and wages are 7.99.
0.49
The %age change = 7.5 100 = 6.53%
In 2005, the sale = 195.55 and salaries and wages are 8.6.
0.61
7
Hence the %age change = .99 100 = 7.63%
Hence, the maximum growth is in 2001.
105. Given x = 10 Going by the options

Option (1) x = 10, y


=5
5
10
3
4
6
8
Satisfies
9
2

Option (2) x
= 10, y = 7
7
10
5
6
8
8
9

Sum
of
Diagonals
=7+8+9
9+8+5

Option (3) x = 10, y


=9
9
10
7
8
10
8
9

11

Sum of Diagonals
= 9 + 10 + 11 9 + 10 +
7

106. Sum of top row = y + x + y 2 = 2y + x 2


If 2y + x 2 = 21
Going by option

Option (1) x = 5, 2y
+ 5 2 = 21 y = 9
i.e. x = 5, y = 9
9
5
7
8
10
3 9+

Option (2) If x = 7 y
=8

10 + 11 21
4
6
11

9 + 10 21
6
5

8
7

Option (3) If x = 9, y = 7
7
9
5
7 7 + 8 + 9 21
6
8

7
9

6
5
10

8+

` 46
8

112. X1 = 5, x2 = 25, x3 = GCD (5, 25) + 5 = 5 + 5 = 10

Hence, Option 4.
107. The equation of the two circles can be rewritten as
S1 = (x 1)2 + (y 3)2 = r2
S2 = (x 4)2 + (y + 1)2 = 32
The two circles will intersect at two points
(i) If Distance of Centres < Sum of Radii
(ii) If Distance of Centres > Difference of Radii
i.e.

(1 4) 2 (3 1) 2 r 3 r > 2 and 5 > r 3

r<8
Combining 2 < r < 8.
108. As per the given condition
Dividend = Divisor Quotient + 41
Divisor Quotient = 1956. Also the divisor has to be
greater the 41
Clearly there is only one number satisfying

1956 2 2 31 1631.

Thus the possible divisors are 163, 163 2, 163 22, 163
3, 163 2 3 and 163 22 3.
Hence there are 6 natural numbers.
109. For positive real numbers, A.M G.M
c c

a
b cc

2
a b
c c
c
2
a b
ab
2

4c 2
c c

ab
a b
2

4 a 2 b2
c c

ab
a b

x2
25

x3
10

x4
30

x5
20

x6
40

x7
40

x8
80

x9
80

x10
160
x20

x1

x1

x1

x1

x1

x1

x1

x1

15

16
0

32
0

32
0

64
0

64
0

12
80

12
80

25
60

25
60

51
20

Hence, Option 4.
113. (i) F(0, 0) = 0
(ii) F(2x, 2y) = F(2x + 1, 2y + 1) = F(x, y)
(iii) F(2x, 2y + 1) = F(2x + 1; 2y) = F(x, y) + 1
F(11, 7) = F(2 5 + 1, 2 3 + 1) = F(5, 3)
F(5, 3) = F(2 2 + 1, 2 1 + 1) = F(2, 1)
F(2, 1) = F(2 1, 2 0 + 1) = F(1, 0) + 1
F(1, 0) = F(2 0 + 1, 2 0 + 0) = F(0, 0) + 1
F(11, 7) = F(1, 0) + 1 = F(0, 0) + 1 + 1 = 2. Hence, Option 1.
114. A.T.Q. probability of getting 3black balls=4/106/12/8/14
=4/35 Answer is option 4
115. Here P = L2(30 L) (1 + 2L)2
L = 6 P = 36(30 6) (1 + 12)2
= 36 24 169 = 146016
L = 5 P = 25(30 5) (1 + 10)2
= 25 25 121 = 75625
L = 4 P = 16(30 4) (1 + 8)2
= 16 26 81 = 33696. Thus for Maxm. P, L =
6. Hence, Option 2.
116 20 Electronic Components 5 Defective
15 Good
117.
5
P(A) = Ist Component Defective =

20

c c
a b
4 ................(i)
a b
b a

a b

b a ab
Now again
2
b a

P(B) = 2nd Component is Defective.


Now 2nd Component is defective (Probability) is dependent
on whether Ist drawn is defective OR Ist drawn is good.
P(IInd defective/Ist drawn is Good) =

5
19

4
19
5 4 15 5
95
1
Reqd. Prob. =

20 19 20 19 20 19 4

P(IInd defective/Ist drawn is defective) =

a b
2
b a
Put this value in (i), we get

x1
5

c c
c c
4 2 8
a b
a b
Hence, the minimum value is 8.
110. Assign any arbitrary value to a. Let a = 11 aC2 = 11C2
= 55 b = 55
Now 55C2 = 55.27. Option (2) = 3 a + 1C4 = 3 12C4 =
27 55.
111. Let the original length, breadth and height be l, b and h
respectively. Original volume = lbh. New length = 125l/100
= 5l/4. New breadth = 4b/3. Let new height be x cm. New
volume is 4/3 times original volume. New volume = 5l/4
4b/3 p = 4/3. lbh 5/4 4/3 lbx = 4/3 lbh p = 4/5 h.
So the height decreases by 20%. So option 2 is the right
answer.

116. A and B are dependent. Hence, Option 2.


117. P(A) = P(B). Hence, Option 3.
118. Find the amount if Rs. 25000 is given @ 12% p.a. for 2 years
i.e. 25000 112/100 112/100 = Rs. 31360. Now out of this
the sum of money paid is 1000 24 = 24000. The balance
left is 31360 24000 = Rs. 7360.
119. For the equation X2 3ax + b = 0 to have equal roots
Disc. = 0 (-3a)2 4 1 b = 9a2 4b = 0 4b = 9a2.
Hence, Option 4.
120. We know a b a b . So

| z 1 z | | z | | 1 z | | z 1|
| z | | z 1 | 1 . Therefore the minimum value of
| z | | z 1 | 1

121. f(x) = x4 6x2, f(x) = 4x3 12x. Now f(x) = 0 4x3


12x = 0

` 47
4x(x2 3) = 0 x = -, x =

3,x= 3

Thus Maximum value of =

3 x 0 f ' ( x ) ive

Case I

128. The possibilities arise


D, E, C, A, B, G, F
or C, A, B, G, F, D, E
It is not possible to identify relation between E & F. Neither
of the statements are sufficient.
129.

So f(x) is decreasing
Case II 0 <x < 3 f(x) = +ive
So f(x) is increasing in (0,
Case III x >

3)

3 f(x) = +ive

So f(x) is increasing in ( 3 , ). Hence, Option 1.


122. Q1. TSA of tent = TSA of triangular prism (except one
lateral face) + TSA of cuboid (except top and bottom face) =

3
[2 4

(24) 2 2 24 30 2 15(30 24)

3060 288 3

99 1000
18.33 hr.
5400
Hence, Option 2.
125. For minimizing the expression numerator should be
minimum and denominator should be maximum
So a should be = 1, b =8 and c =9

abc
189

10.5
a b c 1 8 9
. Hence, Option 4.
126. Now Under 15, there are 200000 females, in that carriers are
given to be 0.2 in each category. Multiplying that gives you
200000 0.2 = 40000. Between 15 25, there are 80000
females, in that carriers are given to be 0.1 in each category.
Multiplying that gives you 80000 0.1 = 8000. Over 35,
there are 420000 females, in that carriers are given to be 0.1.
Multiplying that gives you 420000 0.1 = 42000. Thus the
total is 40000 + 8000 + 42000 = 90000. Thus 3rd option.
127. A = x3y + xy3 = xy(x2 + y2). A = xy(1 xy)
Let xy = , A = 2 + - = 0

The roots are

roots 1 - 4 0

1
4

AP PQ
ST PQ
4.5 BQ PQ

].

Hence answer is 4th option.


123. In this case we find all two digit, three digit, four and five
digit palindromic numbers.
A two digit palindromic number is of the form xx. Since x
can be selected in 9 different ways, i.e 1 to 9, 9 two digit
palindromes can be formed.
A three digit palindrome is of the form xyx. x can be selected
in 9 different ways and y can be selected in 10 different
ways (0 to 9). So number of three digit palindromes = 9 10
= 90.
Four digit palindromes are of the form xyyx and five digit
palindromes are of the form xyzyx. In each of the given
cases, x can take 9 values, y and z can take 10 values each.
So number of four digit palindromes = 90 and number of five
digit palindromes = 900. So total number of palindromes
between 10 and 100000 = 900 + 90 + 90 + 9 = 1089. So
answer is option 3.
124. Reqd. volume = 6 10 1/2(.8 + 2.5)= 99 m3
Flow rate = 90 lit/min = 5400 lit/hr. Reqd. time =

1 1 4

1
4

. For the equation to have real

Triangle PST and PBQ are similar so PT/PQ = ST/BQ ----(1)


Triangle QST and QAP are similar so QT/QP = ST/AP ----(2)By dividing two equations
PT/QT = AP/BQ = 6/4.5 = 4/3 If PQ = 8 then we can get
value of ST from equation 1 so answer should be option 1
and second alone is not sufficient
130.

C
2

E
R

P
A

BPE and BRC are similar.

BR BC

BP 3 cms PQ = 6 cm.
BP BE
Both the statements are required.
131. f(x, y) = (x y)2 x2y2.
(x y) is a factor ; f(x, y) is symmetric ; Both are
required.
132. Because of a 20% increase in speed, the time required to
travel decreases by 16 2/3%, so the 15 min he could cover up
by increasing the speed is basically 1/6 th of the usual time.
He usually takes 1 hours to travel from house to his office.
By increasing his speed by 1.5 kmph, the distance he usually
covers in 90 min is covered in 72 min. So

V
72

V 1.5 90

V = 6 kmph. So distance from house to office = 1.5 6 =


9 km. So answer is 2nd option.

` 48

Answer keys & Explanations Analytical Reasoning


1.
2.
3.
4.
5.
6.
7.
8.
9.
10.
11.
12.
13.
14.
15.
16.
17.

1.

2.

3
4
2
1
3
3
1
4
5
1
3
3
1
3
3
3
5

5
5
4
2
3
4
1
1
2
5
4
4
5
5
3
1
4

18.
19.
20.
21.
22.
23.
24.
25.
26.
27.
28.
29.
30.
31.
32.
33.
34.

In order to find the profit /loss, we require the information


about initial costs and the distribution of costs. As from
statement I, we can find the distribution of sales of all the
three retail formats. Now from statement II, the
information about the initial costs is given and from the
given tables, we already have the distribution of
additional costs as well as the ratio of units consumed. So
on combining both the statements, we can easily find the
profit/loss of different retail formats.
For Online :
Sales = 50% of 60000 = 30000 ( as given above)
Initial costs = 50% of 39000 = 19500 (as given above)
Contribution = 30000-19500 = 10500
Additional costs :

60
20
= 1200, Y = 3100
= 620, G =
275
100
21
21
= 1260, V = 2400
=630
4800
80
80
R = 5500

Total = 3710
Therefore, Loss = 3710 3500 = 210
For Bookstore :
Sales = 2/3 of 30000 = 20000 ( as given above)
Initial costs = 2/3 of 19500 = 13000 (as given above)
Contribution = 20000-13000 = 7000
Additional costs :

10
30
= 200, Y = 3100
= 930,
275
100
9
9
= 540, V = 2400
=270
G = 4800
80
80

R = 5500

200
50
= 4000, Y = 3100
= 1550, G =
R = 5500
275
100
50
50
= 3000, V = 2400
=1500
4800
80
80
3.

Total = 10050. Therefore, Profit = 10500 10050 = 450.


For Online :
Sales = 50% of 60000 = 30000 ( as given above)
Initial costs = 50% of 39000 = 19500 (as given above)
Contribution = 30000-19500 = 10500
Additional costs :

200
50
= 4000, Y = 3100
= 1550, G =
275
100
50
50
= 3000, V = 2400
=1500
4800
80
80
R = 5500

Total = 10050
Therefore, Profit = 10500 10050 = 450
For Supermarket :
Sales = 1/3 of 30000 = 10000 ( as given above)
Initial costs = 1/3 of 19500 = 6500 (as given above)
Contribution = 10000-6500 = 3500
Additional costs :

4.

Total = 1940
Therefore, Profit = 7000 1940 = 5060
So clearly Supermarket is making least profit, as it in loss.
For Online :
Sales = 50% of 60000 = 30000 ( as given above)
Initial costs = 50% of 39000 = 19500 (as given above)
Contribution = 30000-19500 = 10500
Additional costs :

200
50
= 4000, Y = 3100
= 1550, G =
275
100
50
50
= 3000, V = 2400
=1500
4800
80
80
R = 5500

Total = 10050
Therefore, Profit = 10500 10050 = 450
For Supermarket :
Sales = 1/3 of 30000 = 10000 ( as given above)
Initial costs = 1/3 of 19500 = 6500 (as given above)
Contribution = 10000-6500 = 3500

` 49
Additional costs :

60
20
= 1200, Y = 3100
= 620, G =
R = 5500
275
100
21
21
= 1260, V = 2400
=630
4800
80
80
Total = 3710
Therefore, Loss = 3710 3500 = 210
For Bookstore :
Sales = 2/3 of 30000 = 20000 ( as given above)
Initial costs = 2/3 of 19500 = 13000 (as given above)
Contribution = 20000-13000 = 7000
Additional costs :

10
30
= 200, Y = 3100
= 930, G =
275
100
9
9
= 540, V = 2400
=270
4800
80
80

811

8.
9.

5.

6.

7.

10.

12.

13.

14.

15.

Against
Karn

Karn/Bhim

Recreation Bill

Arjun, Bhim

Karn

School Bill

Karn

Arjun, Bhim

Tax Bill

Karn

Bhim, Arjun

Bill

For

Against

Recreation Bill

Arjun, Bhim

Karn

School Bill

Bhim

Arjun, Karn

Tax Bill

Karn

Bhim, Arjun

For

Against

Recreation Bill

Arjun, Bhim

Karn

School Bill

Karn

Arjun, Bhim

Tax Bill

Karn

Bhim, Arjun

Or
Bill

11.

For
Arjun,
Bhim

Arjun and Either Karn


or Bhim
Bhim and Either Karn
Tax Bill
Karn/Arjun
or Arjun
It is clear from the table that Karn and Arjun may vote for
two Bills.
Bill
For
Against
School Bill

R = 5500

Total = 1940
Therefore, Profit = 7000 1940 = 5060. Clearly,
Bookstore is making the highest profit.
Since students sitting next to each other have marked
same answers, thus we get 2 possibilities i.e.
1. They have marked correct answers
2. They have marked incorrect answers
Now, in case of correct answers, teacher cannot detect
cheating, however, students would have cheated
definitely if most of the closely seated students have same
incorrect answers. Thus correct option is 3.
Here I, II, II, IV statements are required to know the
strength of the class i.e. Strength order is A>B>C>D, also
Strength of C is greater than 2 from statement VI. From
statement V, ABCD = 10x, which implies that one of
A, B, C, D is 5. Taking the value of x from 1-11, we
either get the less factors or the number of students come
out to be same. When x = 12, we get the factors to be 2, 3,
4, 5. So these numbers satisfy the given statements and
give us the strength of all the sections. Thus we require
only first 6 statements to find the strength of the sections.
From 1st guess, the ranking is A, B, C, D, E but this
ranking is wrong which means A doesnt precede B in
correct ranking, B doesnt precede C, C doesnt precede
D and D doesnt precede E.
From 2nd guess, the ranking is D, A, E, C, B but in this
ranking 2 positions are correct and 2 students in ranking
followed their immediate predecessors
Now if we consider option 1 to be correct, it will imply
that both A and E are at incorrect positions and out of B,
C and D one more should be incorrect. Secondly, E will
either come at position I or IV, because if its on II, it will
make 3 positions in 2nd guess correct, if its on V it will
negate the incorrectness of 1st guess.
So we get two possible rankings i.e.
E, D, A, C, B and D, C, A, E, B
In first case, we have two predecessors at their correct
positions according to 2nd guess but in 2nd case, only one
predecessor is at their correct position.
Thus option 1 is correct.

Bill
Recreation
Bill

Clearly. option 1 is the answer as in the question, it is


asked that which of the options COULD be true.
And the rest of the options are never possible.
Bill
For
Against
Recreation Bill

Arjun, Bhim

Karn

School Bill

Karn

Arjun, Bhim

Tax Bill

Karn

Bhim, Arjun

The criteria Mr. Dev and his family are looking for are: a)
Long term service and trustworthy b) Age and
experience. Based on this Mr. Dev would like to recruit
Bal Singh.
The criteria Mr. Dev is looking for are: a) Long term
service and trustworthy and His family is looking for a)
Age and experience. These criteria are met in option 1
only. Option 2 is ruled out because Chintan satisfies the
condition family is looking for so cant be least preferred.
Similarly Bal Singh cant be rejected by HR has he has
experience.
Criteria G.M (H.R) is looking for is that the new driver
does not lead to discontent among the senior employees
of the firm. Looking into the case provided, Ram Singh
was criticized by senior employees as he provided
information about competitors to DEV. And since
Chethan is also working with the competitor, hence may
create similar discontent among senior employees.
M3, M5, M1 or M2 or M4 will get rooms as per their best
preference. So, there are 3 managers and answer is (c)
option.

` 50
16.

301

302

303

304

305

M1

M2

M3

M4

M5

SE
E

21.

3
E

NE

304

305

M1

M2

M3

M4

M5

DESTINATION

AGE

PROFESSION

CITY

18

NAME

23.
24.

Teacher
Koderma

Kochi

<36 = 31+4
= 35

Teacher

Chennai

<36 = 35

Engineer

New
Delhi

Mumbai

<34

Teacher

Mughal
Sarai

Hyderabad

31
(Youngest)

Doctor

Kanpur

18.

Can be checked from the above table.

19.

Since 3 persons are from Army background so the


remaining 3 must have joined as freshers. Since the
question discusses about the persons from the same
profession, the best possible option will be 5th .
Can be checked from the above table.
Based on the input the arrangements of the room on both
the floors is as follows

25.

26.

3
1

2
1

(Ground Floor)

Sum of Preference = 3 + 1 + 1 + 2 + 2 = 9
Hence, Answer is 5th option.

(1st Floor)

Based on the inputs the distribution is as


1

Engineer

N
NW
(Ground floor)

303

Oldest

302

Kolkata

SW

301

Bangalore

2
3

22.

The number of students is 27 and its distribution is as


indicated
SE

17.

S
W

So, minimum total Preference = 2 + 1 + 2 + 1 = 7


So, Answer is 3rd option.

20.
21 22

2
3

1
3

(1st Floor)

By options.
Option (1) might be true
Suppose one person sees the Guru Dutt film. Then, there
would be minimum two persons to see the Satyajit Ray
film.
Therefore, option (2) is incorrect.
Vyomkesha and Yogi see the same film. Therefore,
option (3) is incorrect.
If exactly two film buffs see the Ritwik Ghatak film,
then there would be five film buffs to see the Guru Dutt
or Satyajit Ray films.
It is given that exactly twice as many of the film buffs
see the Satyajit Ray film as they see the Guru Dutt film.
Therefore, option (4) is incorrect. Similarly, option (5)
could not be correct.
If one film buff sees the Ritwik Ghatak film, then two
film buffs would see Guru Dutt film and four film buffs
would see Satyajit Ray film.
Vyomkesha, Yogi, Lalatakshaya, Indra/Maheshwar
Satyajit Ray film
Gangadhar Ritwik Ghatak Film
Rudra, Indra or Maheshwar Guru Dutt film.
If Vyomkesha and Gangadhar see the same film, then
Yogi would also be there. Thus,
Vyomkesha, Yogi, Gangadhar Ritwik Ghatak film
Three persons cannot see Guru Dutt film. Therefore
option (1) is incorrect.
If Gangadhar sees the Ritwik Ghatak film, then Rudra

` 51

27.

28.

2931.

could not see the Ritwik Ghatak film. Therefore, option


(3) is incorrect.
Vyomkesha cannot see the Satyajit Ray film because
Gangadhar sees wither Guru Dutt film or Ritwik Ghatak
film. Therefore, option (4) is incorrect. For similar
reason, option (5) is also not correct.
Vyomkesha and Yogi could not see the Guru Dutt film
because in that case Gangadhar would see the Ritwik
Ghatak film and Indra and Maheshwar would see the
Satyajit Ray film. But it is given that Indra and
Maheshwar do not see the same film.
If A is a thief, then B is not a guard. However, E claims
that A is a guard. Then B must be a guard. Now C is a
thief and D is a guard. And, hence E must be a thief.
Therefore, E cannot be a guard and he is telling lie. Thus,
there is only one guard, i.e., C and four thieves.
V > Y, Q
.. (i)
Z>Q

...(ii)

R>T>V

...(iii)

V>S

...(iv)

R>W

....(v)

From equations (i) and (iii),


R > T > V > Y, Q
Now,

R T V Y, Q
Anywhere could be Z

R T V Y, Q
R T V Y, Q
and
.
S
W
29.

30.

In no case Y would be the second product to be launched


option (1) X
In no case R would be the third product to be launched
option B (X)
Q could be fifth but in no case it would be fourth
option C (X)
V cannot be sixth product to be launched as Y, Q and S
are launched after V option E (X)
Therefore, option 4 is the answer

ZRTV

Y, Q
S
W

S cannot be launched before V and hence not before T.


W could be launched before T.
W could be launched after V.
Y could be launched after Q.
Z was certainly launched before W.
31.

RTVZQ
Y
S
W
R is the first product to be launched.
T could be the second or third product to be launched.
Q could be the fifth or eighth product to be launched.
V could be the third or fourth product to be launched.
Z may be the sixth product to be launched.

3234.

Hence, answer is option 5.


Mobile
Digital
Phone
Camera

Music
Player

Office
Document
Viewer

T
V

32.
33.
34.

T may have any of three options.


Z may have Music Player and either Digital Camera or
Office Document Viewer.
It is clear from the table that it is possible to determine
the option(s) of phones V, W, X and Y.
Three or four mobile phones feature a music player.
According to the conditions given in the question, the
information may be put as:
Mobile
Digital
Music
Office
Phone
Camera
Player
Document
Viewer
T

W and Z have both the options in common.

Indias Fastest Growing Test Prep Portal

You might also like